Você está na página 1de 80

VECTOR

VECTOR & CALCUL


CALCULUS
CULUS
THEORY AND EXERCISE BOOKLET

CONTENTS

S.NO. TOPIC PAGE NO.

♦ VECTOR ....................................................................................................... 2 – 19

♦ CALCULUS ................................................................................................ 20 – 37

♦ EXERCISE - I ............................................................................................. 38 – 52

♦ EXERCISE - II ............................................................................................ 53 – 63

♦ EXERCISE - III ............................................................................................. 64 – 68

♦ EXERCISE - IV ............................................................................................ 69 – 73

♦ EXERCISE - V ............................................................................................. 74 – 76

♦ ANSWER KEY ............................................................................................ 77 – 80

394 - Rajeev Gandhi Nagar Kota, Ph. No. 0744-2209671, 93141-87482, 93527-21564
www. motioniitjee.com , email-hr.motioniitjee@gmail.com

Downloaded from www.iitjeephysics4u.com


Page # 2 VECTOR & CALCULUS

VECTOR
1. SCALAR :
In physics we deal with two type of physical quantity one is scalar and other is vector. Each scalar
quantity has a magnitude and a unit.
For example mass = 4kg
Magnitude of mass =4
and unit of mass = kg
Example of scalar quantities : mass, speed, distance etc.
Scalar quantities can be added, subtracted and multiplied by simple laws of algebra.

2. VECTOR :

Vector are the physical quantites having magnitude as well as specified direction.
For example :
Speed = 4 m/s (is a scalar)
Velocity = 4 m/s toward north (is a vector)
If someone wants to reach some location then it is not sufficient to provide information about the
distance of that location it is also essential to tell him about the proper direction from the initial
location to the destination.
 
The magnitude of a vector ( A ) is the absolute value of a vector and is indicated by | A | or A.

Example of vector quantity : Displacement, velocity, acceleration, force etc.

Knowledge of direction
N

W E

S
3. GENERAL POINTS REGARDING VECTORS :

3.1 Representation of vector :


Geometrically, the vector is represented by a line with an arrow indicating the direction of vector as
Tail Head
Length
(magnitude)
Y →
 A
Mathematically, vector is represented by A .
Sometimes it is represented by bold letter A.
θ

Thus, the arrow in abow figure represents a vector A
X
in xy-plane making an angle θ with x-axis.

A representation of vector will be complete if it gives us direction and magnitude.

394,50 - Rajeev Gandhi Nagar Kota, Ph. No. : 93141-87482, 0744-2209671


IVRS No : 0744-2439051, 52, 53, www. motioniitjee.com , hr@motioniitjee.com

Downloaded from www.iitjeephysics4u.com


VECTOR & CALCULUS Page # 3

   
Symbolic form : v, a,F, s used to separate a vector quantity from scalar quantities (u, i, m)

Graphical form : A vector is represented by a directed straight line,


N
having the magnitude and direction of the quantity represented by it. B
5cm
e.g. if we want to represent a force of 5 N acting 45° N of E Head
(i) We choose direction co-ordinates. 45°
W E
(ii) We choose a convenient scale like 1 cm ≡ 1 N A
(iii) We draw a line of length equal in magnitude and in the direction
tail
of vector to the chosen quantity.
S
(iv) We put arrow in the direction of vector. 1cm ≡1N
AB
Magnitude of vector :

| A B |= 5 N

3.2 Angle between two Vectors (θ


θ)
Angle between two vectors means smaller of the two angles between the vectors when they are
placed tail to tail by displacing either of the vectors parallel to itself (i.e 0 ≤ θ ≤ π).

 
B B

B
θ ⇒ θ
θ   
A A A

       
Ex.1 Three vectors A, B, C are shown in the figure. Find angle between (i) A and B , (ii) B and C , (iii) A

and C .
x x
 30º 45º
A  
B C
30º x

Sol. To find the angle between two vectors we connect the tails of the two vectors. We can shift B &

C such that
   
tails of A,B and C are connected as shown in figure. A
  30º x
Now we can easily observe that angle between A and B is 60º, 30º
45º
     
B and C is 15º and between A and C is 75º. C B

3.3 Negative of Vector


It implies vector of same magnitude but opposite in direction.

→ →
A –A

394,50 - Rajeev Gandhi Nagar Kota, Ph. No. : 93141-87482, 0744-2209671


IVRS No : 0744-2439051, 52, 53, www. motioniitjee.com , hr@motioniitjee.com

Downloaded from www.iitjeephysics4u.com


Page # 4 VECTOR & CALCULUS

3.4 Equality of Vectors.


Vectors having equal magnitude and same direction are called equal vectors

C


A 
B
  
if | A| =|B| =| C|
and A  = B = C 
  
then A = B = C

3.5 Collinear vectors :


Any two vectors are co-linear then one can be express in the term of other.
 
a = λb (where λ is a constant)
3.6 Co-initial vector : If two or more vector start from same point then they called co-initial vector.
A
 
A B
B
C

e.g. C

O  D
D
here A, B, C, D are co-initial.

3.7 Coplanar vectors :


Three (or more) vectors are called coplanar vectors if they lie in the same plane or are parallel to the
same plane. Two (free) vectors are always coplanar.

Important points
 If the frame of reference is translated or rotated the vector does not change (though its components
may change).
or

 
ct

A S'
ve

O'
S
O
Two vectors are called equal if their magnitudes and directions are same, and they represent values of
same physical quantity.

3.8 Multiplication and division of a vector by a scalar


  
Multiplying a vector A with a positive number λ gives a vector (B = λ A ) whose magnitude become λ
 
times but the direction is the same as that of A . Multiplying a vector A by a negative number λ gives
  
a vector B whose direction is opposite to the direction of A and whose magnitude is –λ times | A | .

  1
The division of vector A by a non-zero scalar m is defined as multiplication of A by .
m
 
At here A and B are co-linear vector

394,50 - Rajeev Gandhi Nagar Kota, Ph. No. : 93141-87482, 0744-2209671


IVRS No : 0744-2439051, 52, 53, www. motioniitjee.com , hr@motioniitjee.com

Downloaded from www.iitjeephysics4u.com


VECTOR & CALCULUS Page # 5

  
Ex.2 A physical quantity (m = 3kg) is multiplied by a vector a such that F = ma . Find the magnitude

and direction of F if

(i) a = 3m/s2 East wards

(ii) a = –4 m/s2 North wards
 
Sol. (i) F = ma = 3 × 3 ms −2 East wards

= 9 N East wards
 
(ii) F = ma = 3 × ( −4 ) N North wards

= –12 N North wards


= 12 N South wards

4. LAWS OF ADDITION AND SUBTRACTION OF VECTORS :


4.1 Triangle rule of addition : Steps for additing two vector representing same physical quantity by
triangle law.
(i) Keep vectors s.t. tail of one vector coincides with head of other.
(ii) Join tail of first to head of the other by a line with arrow at head of the second.
(iii) This new vector is the sum of two vectors. (also called reultant)

D
D

CD AD
C
(i) (ii) (iii) AB + CD = AD
A B A C
AB B

Take example here.


Q. A boy moves 4 m south and then 5 m in direction 37° E of N. Find resultant displacement.

4.2 Polygon Law of addition :


This law is used for adding more than two vectors. This is extension of triangle law of addition. We
keep on arranging vectors s.t. tail of next vector lies on head of former.
When we connect the tail of first vector to head of last we get resultant of all the vectors.


d 
 
a+  c + d

d
b+ 
+b+

c
P = a 

 
c  c
 +b
a 



b  b
a

a

        
( ) ( )
Note : P = (a + b ) + c + d = (c + a + d) + d [Associative Law]

394,50 - Rajeev Gandhi Nagar Kota, Ph. No. : 93141-87482, 0744-2209671


IVRS No : 0744-2439051, 52, 53, www. motioniitjee.com , hr@motioniitjee.com

Downloaded from www.iitjeephysics4u.com


Page # 6 VECTOR & CALCULUS

4.3 Parallelogram law of addition :


Steps :
(i) Keep two vectors such that there tails coincide.
(ii) Draw parallel vectors to both of them considering both of them as sides of a parallelogram.
(iii) Then the diagonal drawn from the point where tails coincide represents the sum of two vectors,
with its tail at point of coincidence of the two vectors.


b

a

D a
   C
b b b 
(i) (ii) (iii)
 
  b AC = a + b
a a A 
a B

       
Note : AC = a + b and AC = b + a thus a + b = b + a [Cummutative Law]
Note : Angle between 2 vectors is the angle between their positive directions.
 
Suppose angle between these two vectors is θ, and | a |= a, | b |= b

(AD)2 = (AE)2 + (DE)2 


C a
  D
= (AB + BE)2 + (DE)2  b
b a+
= (a + b cos θ)2 + (b sin θ)2 θ α θ
A 
2 2 2 2 2
= a + b cos θ + 2ab cos θ + b sin θ a B E
= a2 + b2 + 2ab cos θ

Thus,AD = a 2 + b 2 + 2ab cos θ


 
or | a + b |= a 2 + b 2 + 2ab cos θ
angle α with vector a is

DE b sin θ
tan α = =
AE (a + b cos θ)

Important points :
 To a vector, only a vector of same type can be added that represents the same physical quantity and
the resultant
is also a vector of the same type.

 As R = [A2 + B2 + 2AB cosθ]1/2 so R will be maximum when, cos θ = max = 1,


i.e., θ = 0º, i.e. vectors are like or parallel and Rmax = A + B.
 
 | A |=| B | and angle between them θ then R = 2A cos θ / 2
 
 | A |=| B | and angle between them π – θ then R = 2 A sin θ / 2

 The resultant will be minimum if, cos θ = min = – 1, i.e., θ = 180º, i.e. vectors are antiparallel and Rmin = A – B.

 If the vectors A and B are orthogonal, i.e., θ = 90º, R = A 2 + B 2

394,50 - Rajeev Gandhi Nagar Kota, Ph. No. : 93141-87482, 0744-2209671


IVRS No : 0744-2439051, 52, 53, www. motioniitjee.com , hr@motioniitjee.com

Downloaded from www.iitjeephysics4u.com


VECTOR & CALCULUS Page # 7

 As previously mentioned that the resultant of two vectors can have any value from (A – B) to (A + B)
depending on the angle between them and the magnitude of resultant decreases as θ increases 0º to
180º.

 Minimum number of unequal coplanar vectors whose sum can be zero is three.

 The resultant of three non-coplanar vectors can never be zero, or minimum number of non coplanar
vectors whose sum can be zero is four.

5. SUBTRACTION OF VECTOR :
→ →
Negative of a vector say – A is a vector of the same magnitude as vector A but pointing in a

direction opposite to that of A .
→ → → → → → → →
Thus, A – B can be written as A + (– B ) or A – B is really the vector addition of A and – B .

A


–A

→ → → →
Suppose angle between two vectors A and B is θ. Then angle between A and – B will be 180° – θ as
shown in figure.

B 180° − θ →
α A
β

→ →
A –B → → →
S = A– B
(a)
(b)
→ → →
Magnitude of S = A – B will be thus given by
→ →
S = | A– B | = A 2 + B 2 + 2AB cos(180° – θ)

or S= A 2 + B 2 – 2 AB cos θ ...(i)


For direction of S we will either calculate angle α or β , where,

B sin(180° – θ) B sin θ
tanα = = ...(ii)
A + B cos(180 ° – θ) A – B cos θ

A sin(180° – θ) A sin θ
or tan β = = ...(iii)
B + A cos(180 ° – θ) B – A cos θ
Ex.3 Two vectors of 10 units & 5 units make an angle of 120° with each other. Find the magnitude &
angle of resultant with vector of 10 unit magnitude.
 
Sol. | a + b |= a 2 + b 2 + 2ab cos θ = 100 + 25 + 2 × 10 × 5(–1/ 2) = 5 3

5 sin120° 5 3 5 3 1
tan α = = = = ⇒ α = 30°
10 + 5 cos120° 20 – 5 5 × 3 3
[Here shows what is angle between both vectors = 120° and not 60°]

394,50 - Rajeev Gandhi Nagar Kota, Ph. No. : 93141-87482, 0744-2209671


IVRS No : 0744-2439051, 52, 53, www. motioniitjee.com , hr@motioniitjee.com

Downloaded from www.iitjeephysics4u.com


Page # 8 VECTOR & CALCULUS

   
Note : A – B or B – A can also be found by making triangles as shown in figure. (a) and (b)
→ → → →
B B B –A
→ →
A–B
Or

A →
(a) (b) A

Ex.4 Two vectors of equal magnitude 2 are at an angle of 60° to each other find magnitude of their
sum & difference.
 
Sol. | a + b |= 2 2 + 2 2 + 2 × 2 × 2 cos 60° = 4 + 4 + 4 = 2 3


b

60°

a

 
| a − b |= 2 2 + 2 2 + 2 × 2 × 2 cos 120 ° = 4 + 4 − 4 = 2


b
60° a

 120°
–b

→ → → →
Ex.5 Find A + B and A – B in the diagram shown in figure. Given A = 4 units and B = 3 units.

B

θ = 60°

A
Sol. Addition :

R= A 2 + B 2 + 2AB cos θ

= 16 + 9 + 2 × 4 × 3 cos 60° = 37 units


B sin θ 3 sin 60°
tan α = = = 0.472
A + B cos θ 4 + 3 cos 60°
∴ α = tan–1(0.472) = 25.3°
→ → →
Thus, resultant of A and B is 37 units at angle 25.3° from A in the direction shown in figure.

394,50 - Rajeev Gandhi Nagar Kota, Ph. No. : 93141-87482, 0744-2209671


IVRS No : 0744-2439051, 52, 53, www. motioniitjee.com , hr@motioniitjee.com

Downloaded from www.iitjeephysics4u.com


VECTOR & CALCULUS Page # 9

Subtraction : S = A 2 + B 2 – 2AB cos θ θ →


α A
= 16 + 9 – 2 × 4 × 3 cos 60° = 13 units →
–B
B sin θ
and tan θ =
A – B cos θ
→ → →
3 sin 60° S = A– B
= = 1.04
4 – 3 cos 60°
∴ α = tan–1 (1.04) = 46.1°
→ → →
Thus, A – B is 13 units at 46.1° from A in the direction shown in figure.

6. UNIT VECTOR AND ZERO VECTOR



Unit vector is a vector which has a unit magnitude and points in a particular direction. Any vector ( A )

can be written as the product of unit vector ( Â ) in that direction and magnitude of the given vector..

 A
A = A  or  =
A
A unit vector has no dimensions and unit. Unit vectors along the positive x-, y-and z-axes of a
rectangular coordinate system are denoted by î, ĵ and k̂ respectively such that | î | = | ĵ | = | k̂ | = 1.
y

ˆj
î x

z
A vector of zero magnitude is called a zero or a null vector. Its direction is arbitrary.

Ex.6 A unit vector along East is defined as î . A force of 105 dynes acts west wards. Represent the
force in terms of î .

Sol. F = −10 5 î dynes
7. RESOLUTION OF VECTORS
  
If a and b be any two non-zero vectors in a plane with different directions and A be another vector
 
in the same plane. A can be expressed as a sum of two vectors-one obtained by multiplying a by a

real number and the other obtained by multiplying b by another real number..
  
A = λa + µb (where λ and µ are real numbers) → →
 b A
We say that A has been resolved into two component vectors namely →
  λ b

A = λa + µb (where λ and µ are real number) → →
 a λ a
We say that A has been resolved into two component vectors namely
 
λa and µb
→ → → →
λ a and µ b along a and b respectively. Hence one can resolve a given vector into two component
vectors along a set of two vectors – all the three lie in the same plane.

394,50 - Rajeev Gandhi Nagar Kota, Ph. No. : 93141-87482, 0744-2209671


IVRS No : 0744-2439051, 52, 53, www. motioniitjee.com , hr@motioniitjee.com

Downloaded from www.iitjeephysics4u.com


Page # 10 VECTOR & CALCULUS

7.1 Resolution along rectangular component : y


It is convenient to resolve a general vector along axes of a
rectangular coordinate system using vectors of unit ĵ
magnitude, which we call as unit vectors. î, ĵ, k̂ are unit
O x
along x, y and z-axis as shown in figure below :


z
7.2 Resolution in two Dimension

Consider a vector A that lies in xy plane as shown in figure,
   y
A = A1 + A 2
   
A 2 = A y ĵ →
A 1 = A x î, A 2 = A y ĵ ⇒ A = A x î + A y ĵ A
Ay = A sin θ
The quantities Ax and Ay are called x-and y-components
θ

of the vector A .  x
A1 = A x î
Ax is itself not a vector but A x î is a vector and so it A y ĵ . A x = A cos θ

Ax = A cos θ and Ay = A sin θ


It's clear from above equation that a component of a vector can be positive, negative or zero

depending on the value of θ. A vector A can be specified in a plane by two ways :

(a) its magnitude A and the direction θ it makes with the x-axis; or

–1
Ay
(b) its components Ax and Ay A= A 2x + A 2y , θ = tan
Ax

Note : If A = Ax ⇒ Ay = 0 and if A = Ay ⇒ Ax = 0 i.e.,


y Bx
components of a vector perpendicular to itself is always Ax By
zero. The rectangular components of each vector and those 
    B
of the sum C = A + B are shown in figure. We saw that Ay A  Cy
C

   Cx
C = A + B is equivalent to both
Cx = Ax + Bx x
(a)
and Cy = Ay + By
Refer figure (b)

y
Vector R has been resolved in two axes x and y not
perpendicular to each other. Applying sine law in the triangle Ry
Ry R
shown, we have
β
R R Ry β
= x = α
sin[180° – ( α + β)] sin β sin α x
O Rx
(b)
R sin β R sin α
or Rx = and Ry =
sin(α + β) sin(α + β)
If α + β = 90°, Rx = R sin β and Ry = R sin α

394,50 - Rajeev Gandhi Nagar Kota, Ph. No. : 93141-87482, 0744-2209671


IVRS No : 0744-2439051, 52, 53, www. motioniitjee.com , hr@motioniitjee.com

Downloaded from www.iitjeephysics4u.com


VECTOR & CALCULUS Page # 11

Ex.7 Resolve the vector A = A x î + A y ĵ along an perpendicular to the line which make angle 60° with x-
axis.
y
Ay Aycos30°

line y (Aycos30° + Axcos60°)

Axcos60°
Aysin30° 30°
60° x
Sol. Ax x ⇒
(Axsin60° – Aysin30°)

Axsin60°
so the component along line = |Aycos30° + Axcos60°|
and perpendicular to line = |Axsin60° – Aysin30°|

Ex.8 Resolve a weight of 10 N in two directions which are parallel and perpendicular to a slope in-
clined at 30° to the horizontal
Sol. Component perpendicular to the plane
W⊥ = W cos 30°
W||
3 30° W⊥
= (10)
= 5 3 N Ans.
2
and component parallel to the plane W=10 N
30°

 1
W|| =W sin 30° = (10)   = 5 N
2

Ex.9 Resolve horizontally and vertically a force F = 8 N which makes an angle of 45° with the hori-
zontal.

Sol. Horizontal component of →


F is Fv
 1 
FH = F cos 45° = (8)   = 4 2 N
 2 →
F

and vertical component of F is
45°
FH
 1 
Fv = F sin 45° = (8)   = 4 2 N Ans.
 2
8. PROCEDURE TO SOLVE THE VECTOR EQUATION
  
A = B + C ...(1)
(a) There are 6 variables in this equation which are following :

(1) Magnitude of A and its direction

(2) Magnitude of B and its direction

(3) Magnitude of C and its direction.

394,50 - Rajeev Gandhi Nagar Kota, Ph. No. : 93141-87482, 0744-2209671


IVRS No : 0744-2439051, 52, 53, www. motioniitjee.com , hr@motioniitjee.com

Downloaded from www.iitjeephysics4u.com


Page # 12 VECTOR & CALCULUS

(b) We can solve this equation if we know the value of 4 variables [Note : two of them must be directions]
(c) If we know the two direction of any two vectors then we will put them on the same side and other on
the different side.
For example
  
If we know the directions of A and B and C' s direction is unknown then we make equation as
follows:-
  
C = A –B
(d) Then we make vector diagram according to the equation and resolve the vectors to know the
unknown values.

Ex.10 Find the net displacement of a particle from its starting point if it undergoes two sucessive
 
displacement given by S1 = 20m , 37° North of West, S 2 = 50m , 53° North of East

N S
S1 N y
50
S1 20 W x' x
θ =
Sol. 37° 53° E
W E
S y'
S

  
S = S1 + S2
Sx = S1 x + S2 x
Sy = S1 y + S2 y

= – 20 cos 37° + 50 cos 53° = 20 sin 37° + 50 sin 53°


= 14 = 52

S = S 2x + S2y = (14) 2 + (52)2 = 53.85


Angle from west - east axis (x- axis)
Sy 52 26
tanθ = = =
Sx 14 7

 26 
θ = tan–1  
7
   
Ex.11 Find magnitude of B and direction of A . If B makes angle 37° and C makes 53° with x axis and
    
A has magnitude equal to 10 and C has 5. (given A + B + C = 0 )

   C
Sol. –A = C + B y'
 

A = Axi + Ay j  53° B
37°
 x
⇒ – A = – A i + – A j
x y x'

Ax = –(Bcos 37° + Ccos 53°)


y
Ay = –(B sin 37° + C cos 53°)
 2 2
| A|2 = A x + A y
2 2
 4 3  3 4
A2 =  B × + C ×  +  B × + C × 
 5 5  5 5

394,50 - Rajeev Gandhi Nagar Kota, Ph. No. : 93141-87482, 0744-2209671


IVRS No : 0744-2439051, 52, 53, www. motioniitjee.com , hr@motioniitjee.com

Downloaded from www.iitjeephysics4u.com


VECTOR & CALCULUS Page # 13

2 2
 4B   3B 
102 =  + 3 +  + 4
 5   5 

16 2 9 2  3 × 4 4 × 3
⇒ 100 = B + B + 25 + 2 + B
25 25  5 5 

2 48
⇒ B + B – 75 = 0
5
B = 5 (magnitude can not be negative)
& Angle made by A

20
⇒ Ax = –( + 3) = –12
5

15
Ay = –( + 4) = –7
5

Ay –7
tan θ = =
Ax –12

θ = 180° + 25° = 205°

Ex.12 Find the magnitude of F1 and F2. If F1, F2 make angle 30° and 45° with F3 and magnitude of F3 is
  
10 N. (given F1 + F2 = F3 )

Sol. |F3 | = F1 cos 30°+F2 cos 45° F3
& F2 sin 45° = F1 sin 30°
F1
F2
3F1 F2 F2 F
⇒ 10 = + , = 1 45° 30°
2 2 2 2

20 20 2
⇒ F1 = & F2 =
3 +1 3 +1

9. SHORT - METHOD

A   
C = A +B

α
Asinα β 
B
Bsinβ

   
If their are two vectors A and B and their resultent make an anlge α with A and β with B .
then A sin α = β sin β
 
Means component of A perpendicular to resultant is equal in magnitude to the component of B
perpendicular to resultant.

394,50 - Rajeev Gandhi Nagar Kota, Ph. No. : 93141-87482, 0744-2209671


IVRS No : 0744-2439051, 52, 53, www. motioniitjee.com , hr@motioniitjee.com

Downloaded from www.iitjeephysics4u.com


Page # 14 VECTOR & CALCULUS

  
Ex.13 If two vectors A and B make angle 30° and 45° with their B

resultant and B has magnitude equal to 10, then find

magnitude of A . 60°
30° 
So B sin 60° = A sin 30° Bsin60° A
⇒ 10 sin 60° = A sin 30°
A sin 30°
⇒ A = 10 3
  
Ex.14 If A and B have angle between them equals to 60° and their resultant make, angle 45° with A
 
and A have magnitude equal to 10. Then Find magnitude of B .
Sol. here α = 45° and β = 60° – 45° = 15°    
B C = A +B
so A sin α = B sin β
10 sin 45° = B sin 45°

10
So B = sin15° 60°
2
45° 
A
10 1 – cos(2 × 15) 5
= = 2– 3
2 2 2

10. ADDITION AND SUBTRACTION IN COMPONENT FORM :


Suppose there are two vectors in component form. Then the addition and subtraction between these
two are

A = A x î + A y ĵ + A zk̂

B = B x î + B y ĵ + B zk̂

A ± B = ( A x ± B x ) î + ( A y ± B y ) ĵ + ( A z ± B z ) k̂

Also if we are having a third vector present in component form and this vector is added or subtracted
from the addition or subtraction of above two vectors then

C = C x î + C y ĵ + C z k̂

A ± B ± C = ( A x ± B x ± C x ) î + ( A y ± B y ± C y ) ĵ + ( A z ± B z ± C z ) k̂
Note : Modulus of vector A is given by

| A |= A 2x + A 2y + A 2z

→ →
Ex.15 Obtain the magnitude of 2 A – 3 B if

→ →
A = î + ĵ – 2k̂ and B = 2 î – ĵ + k̂

→ →
Sol. 2 A – 3 B = 2( î + ĵ – 2k̂ ) – 3(2 î – ĵ + k̂ )

→ →
∴ Magnitude of 2 A – 3 B = (–4)2 + (5)2 + (–7)2

= 16 + 25 + 49 = 90 Ans.

394,50 - Rajeev Gandhi Nagar Kota, Ph. No. : 93141-87482, 0744-2209671


IVRS No : 0744-2439051, 52, 53, www. motioniitjee.com , hr@motioniitjee.com

Downloaded from www.iitjeephysics4u.com


VECTOR & CALCULUS Page # 15

     
Ex.16 Find A + B and A – B if A make angle 37° with positive x-axis and B make angle 53° with negative

x-axis as shown and magnitude of A is 5 and of B is 10.
 
B A

Sol. 53° 37°

 
for A + B

3 = A sin 37° 8 + 3 = 11
8 = B sin 53°
53°
+ =
–B cos 53° –(6–4) = –2
A cos 37° = 4 = –6

so the magnitude of resultant will be = 112 + (–2) 2 = 5 5

 11
and have angle θ = tan –1   from negative x - axis towards up
 2
 
for A – B

Bcos53°=6 (6+4) = 10
Asin37°=3 + =
53°
–8= –(8–3)
Acos37=4 –Bsin53° = –5

–B

So the magnitude of resultant will be

= 10 2 + (–5) 2 = 5 5

 5
and have angle θ = tan –1   from positive x-axis towards down.
 10 

11. MULTIPLICATION OF VECTORS (The Scalar and vector products) :

11.1 Scalar Product


  
The scalar product or dot product of any two vector A and B , B
   
denoted as A . B (read A dot B ) is defined as the product of θ
their magnitude with cosine of angle between them. Thus,

  A
A.B = AB cos θ (here θ is the angle between the vectos)

394,50 - Rajeev Gandhi Nagar Kota, Ph. No. : 93141-87482, 0744-2209671


IVRS No : 0744-2439051, 52, 53, www. motioniitjee.com , hr@motioniitjee.com

Downloaded from www.iitjeephysics4u.com


Page # 16 VECTOR & CALCULUS

Properties :
• It is always a scalar which is positive if angle between the vectors is acute (i.e.< 90°) and negative if
angle between them is obtuse (i.e., 90° < q ≤ 180°)
   
• It is commutative i.e. A.B = B.A
      
• It is distributive, i.e. A.(B + C) = A.B + A.C
 
  
–1 A.B

• As by definition A . B = AB cos θ . The angle between the vectors θ = cos  
 AB 
 
• A.B = A(B cos θ) = B( A cos θ)
 
Geometrically, B cos θ is the projection of B onto A and vice versa

 
B B


θ θ B
 
A B cos θ A
 
  A.B   
Component of B along A = B cos θ = = Â .B (Projection of B on A )
A

θ
os
 

Ac
  A.B   
Component of A along B = A cos θ = = A.B̂ (Projection of A on B )
B θ

A

• Scalar product of two vectors will be maximum when cos θ = max = 1, i.e., θ = 0°,
 
i.e., vectors are parallel ⇒ ( A.B)max = AB

• If the scalar product of two non-zero vectors vanishes then the vectors are perpendicular.
• The scalar product of a vector by itself is termed as self dot product and is given by
    
( A )2 = A.A = AA cos θ = A2 ⇒ A = A.A
• In case of unit vector n̂ ,

n̂.n̂ = 1 × 1 × cos 0° = 1 ⇒ n̂.n̂ = î.î = ĵ. ĵ = k̂.k̂ = 1

In case of orthogonal unit vectors, î , ĵ and k̂ ; î. ĵ = ĵ.k̂ = k̂.î = 0


 
A.B = ( î A x + ĵ A y + k̂A z ) . ( î B x + ĵ B y + k̂B z ) = [AxBx + AyBy + AzBz]

 
Ex.17 If the vectors P = a î + a ĵ + 3k̂ and Q = a î – 2 ĵ – k̂ are perpendicular to each other. Find the value of
a?
 
Sol. If vectors P and Q are perpendicular
 
⇒ P.Q = 0 ⇒ (a î + a ĵ + 3k̂ ).(a î – 2 ĵ – k̂ ) = 0
⇒ a2 – 2a – 3 = 0 ⇒ a2 – 3a + a – 3 = 0
⇒ a(a – 3) + 1 (a – 3 ) ⇒ a = –1, 3

394,50 - Rajeev Gandhi Nagar Kota, Ph. No. : 93141-87482, 0744-2209671


IVRS No : 0744-2439051, 52, 53, www. motioniitjee.com , hr@motioniitjee.com

Downloaded from www.iitjeephysics4u.com


VECTOR & CALCULUS Page # 17

Ex.18 Find the component of 3 î + 4 ĵ along î + ĵ ?


 
  A.B
Sol. Component of A along B is given by hence required component
B
(3 î + 4 ĵ ).( î + ĵ ) 7
= =
2 2
 
Ex.19 Find angle between A = 3 î + 4 ĵ and B = 12 î + 5 ĵ ?
 
A.B (3 î + 4 ĵ ).(12 î + 5 ĵ)
Sol. We have cos θ = AB =
3 2 + 4 2 12 2 + 5 2
36 + 20 56  56 
cos θ = θ = cos–1  65 
=
5 × 13 65
 
Ex.20 (i) For what value of m the vector A = 2 î + 3 ĵ – 6k̂ is perpendicular to B = 3 î – m ĵ + 6k̂

(ii) Find the component of vector A = 2 î + 3 ĵ along the direction of î + ĵ ?
5
Sol. (i) m = – 10 (ii)
2
Important Note :
Components of b along and perpendicular to a.

Let OA . OB represent two (non-zero) given vectors a, b respectively. Draw BM perpendicular to OA

From ∆OMB, OB = OM + MB

⇒ b = OM + MB B

Thus OM and MB are components of b along a

and perpendicular to a. b
Now OM = ( OM ) â = (OB cos θ) â

= |b| cosθ â = |b| . a . b / |a| |b| . â θ A


O
a M
= a . b / |a| . a/|a| = (a . b) a / |a|2
= (a . b) a / a2
2
MB = b – OM = b – (a . b / |a| ) . a
Hence, components of b along a perpendicular to a are.
(a . b/ |a|2) a and b – (a . b / |a|2) a respectively.

Ex.21 The velocity of a particle is given by v = 3 î + 2 ĵ + 3k̂ . Find the vector component of its velocity parallel

to the line l = î − ĵ + k̂ .
 
Sol. Component of v along l
  
ˆ v.l ˆ v. l 
= v cos θl = v l = 2 l
vl l

(3 î + 2 ĵ + 3k̂ ) ( î − ĵ + k̂ ) 4
= = ( î − ĵ + k̂ )
| î − ĵ + k̂ |2 3

394,50 - Rajeev Gandhi Nagar Kota, Ph. No. : 93141-87482, 0744-2209671


IVRS No : 0744-2439051, 52, 53, www. motioniitjee.com , hr@motioniitjee.com

Downloaded from www.iitjeephysics4u.com


Page # 18 VECTOR & CALCULUS

11.2 Vector product


 
The vector product or cross product of any two vectors A and B , denoted as
   
A × B (read A cross B ) is defined as :
 
A × B = AB sin θn̂
Here θ is the angle between the vectors and the direction n̂ is given by the right - hand - thumb rule.

Right - Hand - Thumb Rule : → → →

  V = A× B
To find the direction of n̂ , draw the two vectors A and B with both the →

tails coinciding. Now place your stretched right palm perpendicular to the n B
  
plane of A and B in such a way that the fingers are along the vector A
 θ
and when the fingers are closed they go towards B . The direction of the

thumb gives the direction of n̂ . A
Properties :
• Vector product of two vectors is always a vector perpendicular to the plane containing the two
vectors i.e. orthogonal to both the vectors → → → →
A and B , though the vectors A and B may or may not be
orthogonal.
→ → → →
• Vector product of two vectors is not commutative i.e. A × B ≠ B× A
→ → → →
But | A× B |=| B× A |= AB sin θ
• The vector product is distributive when the order of the vectors is strictly maintained i.e.
      
A × (B + C) = A × B + A × C
• The magnitude of vector product of two vectors will be maximum when sin θ = max = 1. i.e. θ = 90°
 
| A × B |max = AB
• The magnitude of vector product of two non-zero vectors will be minimum when |sinθ| = minimum = 0,
 
i.e., θ = 0° or 180° and | A × B |min = 0 i.e., if the vector product of two non-zero vectors vanishes, the
vectors are collinear.
• The self cross product i.e. product of a vector by itself vanishes i.e. is a null vector.
  
A × A = AA sin 0°n̂ = 0
 
• In case of unit vector n̂ , n̂ × n̂ = 0 ⇒ î × î = ĵ × ĵ = k̂ × k̂ = 0

• In case of orthogonal unit vectors î, ĵ and k̂ in accordance with right-hand-thumb-rule,

î × ĵ = k̂ ĵ × k̂ = î k̂ × î = ĵ

j j

k
i
i

k
(A) (B)

394,50 - Rajeev Gandhi Nagar Kota, Ph. No. : 93141-87482, 0744-2209671


IVRS No : 0744-2439051, 52, 53, www. motioniitjee.com , hr@motioniitjee.com

Downloaded from www.iitjeephysics4u.com


VECTOR & CALCULUS Page # 19

î ĵ k̂
 
• In terms of components, A × B = A x Ay Az
Bx By Bz
 
A × B = î ( A yB z – A zB y ) + ĵ( A zB x – A xB z ) + k̂( A xB y – A yB x )
   
Ex.22 A is East wards and B is downwards. Find the direction of A × B ?
 
Sol. Applying right hand thumb rule we find that A × B is along North.
     
Ex.23 If A.B =| A × B | , find angle between A and B
   
Sol. A.B =| A × B | AB cos θ = AB sin θ tan θ = 1 ⇒ θ = 45°

 
  A×B  
⇒ n̂ =
Ex.24 A × B = AB sin θn̂ here n̂ is perpendicular to both A and B
AB sin θ
   
Ex.25 Find A × B if A = î – 2 ĵ + 4k̂ and B = 2 î – ĵ + 2k̂

î ĵ k̂
 
Sol. A × B = 1 – 2 4 = î (–4 – (–4)) – ĵ( 2 – 12) + k̂(–1 – (–6 )) = 10 ĵ + 5k̂
3 –1 2

   
Ex.26 (i) A is North-East and B is down wards, find the direction of A × B
   
(ii) Find B × A if A = 3 î – 2 ĵ + 6k̂ and B = î – ĵ + k̂
Ans. (i) North - West. (ii) – 4 î – 3 ĵ + k̂

12. POSITION VECTOR :


Positin vector for a point is vector for which tail is origin & head is the given point itself.
Position vector of a point defines the position of the point w.r.t. the origin.
y

P( x , y )

r

O x

OP = r

r = x î + yĵ
13. DISPLACEMENT VECTOR :
Change in position vector of particle is Q(x2, y2)
known as displacement vector. 
 r2
OP = r1 = x1 î + y1 ĵ
  P( x1, y 1 )
OQ = r2 = x 2 î + y 2 ĵ r1
 
PQ = r 2 − r1 = ( x 2 − x 1 ) î + ( y 2 – y 1 ) ĵ
Th us w e ca n r ep r es ent a vec to r i n sp a ce sta r ti ng fr o m ( x 1 , y 1 ) & end i n g a t
(x2, y2) as (x2 – x1) î + (y2 – y1) ĵ

394,50 - Rajeev Gandhi Nagar Kota, Ph. No. : 93141-87482, 0744-2209671


IVRS No : 0744-2439051, 52, 53, www. motioniitjee.com , hr@motioniitjee.com

Downloaded from www.iitjeephysics4u.com


Page # 20 VECTOR & CALCULUS

CALCULUS
14. CONSTANTS : They are fixed real number which value does not change
Ex. 3, e, a, – 1, etc.

15. VARIABLE :
Somthing that is likly to vary, somthing that is subject to variation.
or
A quantity that can assume any of a set of value.
Types of variables.
(i) Independent variables : Indepedent variables is typically the variable being manipulated or change
(ii) dependent variables : The dependent variables is the object result of the independent variable
being manipulated.

Ex. y = x2
here y is dependent variable and x is independent variable

16. FUNCTION :
Function is a rule of relationship between two variables in which one is assumed to be dependent and
the other independent variable.
The temperatures at which water boils depends on the elevation above sea level (the boiling point
drops as you ascend). Here elevation above sea level is the independent & temperature is the dependent
variable.
The interest paid on a cash investment depends on the length of time the investment is held. Here time
is the independent and interest is the dependent variable.
In each case, the value of one variable quantity (dependent variable), which we might call y, depends
on the value of another variable quantity (independent variable), which we might call x. Since the
value of y is completely determined by the value of x, we say that y is a function of x and represent it
mathematically as y = f(x).

x f(x)
Input f Output
(Domain) (Range)
all possible values of independent variables (x) are called domain of function.
all possible values of dependent variable (y) are called Range of fucntion.
Think of function f as a kind machine that produces an output value f(x) in its range whenever we feed
it an input value x from its domain (figure).
When we study circles, we usualy call the area A and the radius r. Since area depends on radius, we
say that A is a function of r, A = f(r). The eauation A = πr2 is a rule that tells how to calculate a unique
(single) output value of A for each possible input value of the radius r.
A = f(x) = πr2. (Here the rule of relationship which describes the function may be described as square
& multiply by π)
if r=1 A=π
if r=2 A = 4π
if r=3 A = 9π
The set of all possible input values for the radius is called the domain of the function. The set of all
output values of the area is the range of the function.

394,50 - Rajeev Gandhi Nagar Kota, Ph. No. : 93141-87482, 0744-2209671


IVRS No : 0744-2439051, 52, 53, www. motioniitjee.com , hr@motioniitjee.com

Downloaded from www.iitjeephysics4u.com


VECTOR & CALCULUS Page # 21

We usually denote functions in one of the two ways :


1. By giving a formula such as y = x2 that uses a dependent variable y to denote the value of the
fucntion.
2. By giving a formula such as f(x) =x2 that defines a functions symbols f to name the function.
Strictly speaking, we should call the function f and not f(x).
y = sinx. Here the function is y since, x is the independent variable.
4 3
Ex.27 The volume V of ball (solid sphere) of radius r is given by the function V(r) = π(r )
3
The volume of a ball of radius 3m is ?
4
Sol. V(3) = π(3)3 = 36 πm3.
3

Ex.28 Suppose that the function F is defined for all real numbers r by the formula.
F(r) = 2 (r – 1) + 3.
Evaluate F at the input values 0, 2 x + 2, and F(2).
Sol. In each case we substitute the given input value for r into the formula for F:
F(0) = 2(0 – 1) + 3 = – 2 + 3 = 1
F(2) = 2(2 – 1) + 3 = 2 + 3 =5
F(x + 2) = 2 (x + 2 – 1) + 3 = 2x + 5
F(F(2)) = F(5) = 2(5 – 1) + 3 = 11

Ex.29 function f(x) is defined as


f(x) = x2 + 3, Find
f(0), f(1), f(x2), f(x + 1) and f(f(1))
Sol. f(0) = 02 + 3 =3
f(1) = 12 + 3 =4
f(x2) = (x2)2 + 3 = x4 + 4
f(x + 1) = (x + 1)2 + 3 = x2 + 2x + 4
f(f(1)) = f(4) = 42 + 3 = 19

17. DIFFERENTIATION
Finite difference :
The finite difference between two values of a physical is represented by ∆ notation.
For example :
Difference in two values of y is written as ∆y as given in the table below.

y2 100 100 100

y1 50 99 99.5

∆y = y 2 – y 1 50 1 0.5

Infinitey small difference :


The infinitely small difference means very-very small difference. And this difference is represented by
'd' notation insted of '∆'.
For example infinitely small difference in the values of y is written as 'dy'
if y2 = 100 and y1 = 99.9999999999999.....
then dy = 0.00000000000000..........00001

394,50 - Rajeev Gandhi Nagar Kota, Ph. No. : 93141-87482, 0744-2209671


IVRS No : 0744-2439051, 52, 53, www. motioniitjee.com , hr@motioniitjee.com

Downloaded from www.iitjeephysics4u.com


Page # 22 VECTOR & CALCULUS

Definition of differentiation
Another name of differentiation is derivative. Suppose y is a function of x or y = f(x)
Differentiation of y with respect to x is denoted by sumbols f ′ (x)

dy
where f ′(x) = ; dx is very small change in x and dy is corresponding very small change in y..
dx
Notation : There are many ways to denote the derivative of function y = f(x), the most common
notations are these :
Nice and brief and does not name the
y′ "y prime"
independent variable
dy
" dy by dx" Names the variables and uses d for derivative
dx
df
dx
" df by dx" Emphasizes the function's name

d Emphasizes the idea that differentiation is an


f( x ) " d by dx of f "
dx operation performed on f.
Dx f " dx of f " A common operator notation
• One of Newton's notations, now common for time
y " y dot"
derivative i.e. dy/dt

Average rates of change :


Given an arbitrary function y = f(x) we calculate the average rate of change of y with respect to x
over the interval (x, x+ ∆x) by dividing the change in value of y, i.e., ∆y = f(x+ ∆x) – f(x), by length of
interval ∆x over which the change occurred.
The average rate of change of y with respect to x over the interval [x, x + ∆x]

∆y f ( x + ∆ x ) – f ( x )
= =
∆x ∆x
Geometrically
Q
y + ∆y
∆y QR
= = tan θ = Slope of the line PQ
∆x PR
∆y
∆y
In triangle QPR tan θ = P
∆x y θ
R
∆x
therefore we can say that average rate of change of y with
x x + ∆x
respect to x is equal to slope of the line joining P & Q.

The derivative of a fucntion

We know that Average rate of change of y w.r.t x is -

∆y f ( x + ∆x ) – f ( x )
=
∆x ∆x
If the limit of this ratio exists as ∆x → 0, then it is called the derivative of given function f(x) and is
denoted as

dy f ( x + ∆x ) – f ( x )
f ' (x) = = lim
dx ∆x →0 ∆x

394,50 - Rajeev Gandhi Nagar Kota, Ph. No. : 93141-87482, 0744-2209671


IVRS No : 0744-2439051, 52, 53, www. motioniitjee.com , hr@motioniitjee.com

Downloaded from www.iitjeephysics4u.com


VECTOR & CALCULUS Page # 23

18. GEOMETRICAL MEANING OF DIFFERENTIATION :

The geometrical meaning of differentiation is very much useful in the analysis of graphs in physics. To
understand the geometrical meaning of derivatives we should have knowledge of secant and tangent
to a curve.

Secant and Tangent to a Curve

Secant : - A secant to a curve is a straight line, which intersects the curve at any two points.
y
q
Secant

x
Tangent :
A tangent is straight line, which touches the curve a particular point. Tangent is limiting case of secant
which intersects the curve at two overlapping point.

Q
In the figure - 1 shown, if value of ∆x is gradually reduced y + ∆y
then the point Q will move nearer to the point P. If the
process is continuously repeated (Figure-2) value of ∆x
will be infinitely small and secant PQ to the given curve will ∆y
become a tangent at point P.
y P θ R
Therefore ∆x

 ∆y  dy x x + ∆x
 = = tanθ
∆x → 0 ∆x  dx
Figure-1

 dy 
we can say that differentiation of y with respect to x, i.e.   is Q
dx
y + ∆y
equal to slope of the tangent at point P (x,y)
Q

dy Q ∆y
or tanθ =
dx y P θ R
(From fig-1 the average rate change of y from x to x + ∆x ∆x

is identical with the slope of secant PQ) x x + ∆x

Figure-2

Rule No. 1 Derivative Of A Constant


The first rule of differentiation is that the derivative of every constant function is zero.
d
If c is constant, then c=0
dx
d  1
Ex.30
d
dx
(8 ) = 0 , −  = 0 ,
dx  2 
d
dx
( 3)= 0

394,50 - Rajeev Gandhi Nagar Kota, Ph. No. : 93141-87482, 0744-2209671


IVRS No : 0744-2439051, 52, 53, www. motioniitjee.com , hr@motioniitjee.com

Downloaded from www.iitjeephysics4u.com


Page # 24 VECTOR & CALCULUS

Rule No.2 Power Rule


d n
If n is a real number, then x = nx n−1
dx
To apply the power Rule, we subtract 1 from the original exponent (n) and multiply the result by n.

2 3 4
f x x x x …..
Ex.31 2 3
f' 1 2x 3x 4x …..

d  1  d −1 1 d  4  d 12
Ex.32 (i)  = ( x ) = ( −1)x − 2 = − 2 (ii)  3  = 4 ( x −3 ) = 4 ( −3)x − 4 = − 4
dx  x  dx x dx  x  dx x
d 1/ 2 1 1
Ex.33 (a) ( x ) = x −1/ 2 =
dx 2 2 x
Function defined for x ≥ 0 derivative defined only for x > 0
d 1/ 5 1 −4 / 5
(b) (x ) = x
dx 5
Function defined for x ≥ 0 derivative not defined at x = 0

Rule No.3 The Constant Multiple Rule


d du
If u is a differentiable function of x, and c is a constant, then (cu) = c
dx dx

d
In particular, if n is a positive integer, then (cx n ) = cn x n−1
dx

Ex.34 The derivative formula


d
(3x 2 ) = 3 (2x ) = 6x
dx
says that if we rescale the graph of y = x2 by multiplying each y-coordinate by 3, then we
multiply the slope at each point by 3.

Ex.35 A useful special case


The derivative of the negative of a differentiable function is the negative of the function’s derivative.
Rule 3 with c = – 1 gives.
d d d d
( −u) = ( −1.u) = −1 . (u) = − (u)
dx dx dx dx

Rule No.4 The Sum Rule


The derivative of the sum of two differentiable functions is the sum of their derivatives.
If u and v are differentiable functions of x, then their sum u + v is differentiable at every point where
u and v are both differentiable functions in their derivatives.

d d du dv du dv
(u − v ) = [u + ( −1) v ] = + ( −1) = −
dx dx dx dx dx dx
The sum Rule also extends to sums of more than two functions, as long as there are only finite
f u n c t i o n
1
, u2, ........ un are differentiable at x, then so if u1 + u2 + ....... + un, then
s i n t h e s u m . I f u

d du du du
(u1 + u 2 + ...... + un ) = 1 + 2 + ........ + n
dx dx dx dx

394,50 - Rajeev Gandhi Nagar Kota, Ph. No. : 93141-87482, 0744-2209671


IVRS No : 0744-2439051, 52, 53, www. motioniitjee.com , hr@motioniitjee.com

Downloaded from www.iitjeephysics4u.com


VECTOR & CALCULUS Page # 25

4 2
Ex.36 (a) y = x4 + 12x (b) y = x3 + x – 5x + 1
3

dy d 4 d dy d 3 d 4 2 d d
= (x ) + (12 x ) = x +  x − (5 x ) + (1)
dx dx dx dx dx dx  3  dx dx

4
= 4x3 + 12 = 3x2 + . 2x – 5 + 0
3
8
x−5 = 3x2 +
3
Notice that we can differentiate any polynomial term by term, the way we differentiated the polynomials
in above example.

Rule No. 5 The Product Rule


d dv du
If u and v are differentiable at x, then if their product uv is considered, then (uv ) = u +v .
dx dx dx
The derivative of the product uv is u times the derivative of v plus v times the derivative of u. In prime
notation
(uv)’ = uv’ + vu’.
While the derivative of the sum of two functions is the sum of their derivatives, the derivative of the
product of two functions is not the product of their derivatives. For instance,

d d 2
(x . x) = ( x ) = 2x, while d ( x ). d ( x ) = 1.1 = 1 , which is wrong
dx dx dx dx

2
Ex.37 F i n d t h e d e r i v a t i v e s o f y = ( x + 1) (x3 + 3)
Sol. Using the product Rule with u = x2 + 1 and v = x3 + 3, we find

d
[( x 2 + 1)( x 3 + 3)] = (x2 + 1) (3x2) + (x3 + 3) (2x)
dx
= 3x4 + 3x2 + 2x4 + 6x = 5x4 + 3x2 + 6x
Example can be done as well (perhaps better) by multiplying out the original expression for y and
differentiating the resulting polynomial. We now check :
y = (x2 + 1) (x3 + 3) = x5 + x3 + 3x2 + 3

dy
= 5x4 + 3x2 + 6x
dx
This is in agreement with our first calculation.
There are times, however, when the product Rule must be used. In the following examples. We have
only numerical values to work with.

Ex.38 Let y = uv be the product of the functions u and v. Find y’(2) if u(2) = 3, u’(2) = – 4, v(2) = 1, and
v’(2) = 2.
Sol. From the Product Rule, in the form
y’ = (uv)’ = uv’ + vu’,
we have y’(2) = u(2) v’(2) + v(2) u’(2)
= (3) (2) + (1) (–4) = 6 – 4 = 2

394,50 - Rajeev Gandhi Nagar Kota, Ph. No. : 93141-87482, 0744-2209671


IVRS No : 0744-2439051, 52, 53, www. motioniitjee.com , hr@motioniitjee.com

Downloaded from www.iitjeephysics4u.com


Page # 26 VECTOR & CALCULUS

Rule No.6 The Quotient Rule


If u and v are differentiable at x, and v(x) ≠ 0, then the quotient u/v is differentiable at x,

du dv
v −u
d u dx dx
and  =
dx  v  v2

Just as the derivative of the product of two differentiable functions is not the product of their
derivatives, the derivative of the quotient of two functions is not the quotient of their derivatives.

t2 − 1
Ex.39 Find the derivative of y =
t2 + 1
Sol. We apply the Quotient Rule with u = t2 – 1 and v = t2 + 1

dy ( t 2 + 1) 2t − ( t 2 − 1). 2t  d  u  v( du / dt ) − u(dv / dt ) 
=  As   = 
dt ( t 2 + 1)2  dt v v2 

2t 3 + 2t − 2t 3 + 2t 4t
= =
2
( t + 1) 2
( t + 1)2
2

Rule No. 7 Derivative Of Sine Function


d
(sin x ) = cos x
dx

dy d
Ex.40 (a) y = x2 – sin x : = 2x − (sin x ) = 2x – cos x Difference Rule
dx dx

dy d
(b) y = x2 sin x : = x2 (sin x ) + 2x sin x Product Rule
dx dx

= x2cosx + 2x sinx
d
x. (sin x ) − sin x .1
sin x dy dx
(c) y = : = Quotient Rule
x dx x2
x cos x − sin x
=
x2

Rules No.8 Derivative Of Cosine Function


d
(cos x ) = − sin x
dx
Ex.41 (a) y = 5x + cos x Sum Rule
dy d d
= (5 x ) + (cos x ) = 5 – sin x
dx dx dx
(b) y = sin x cos x

dy d d
= sin x (cos x ) + cos x (sin x ) Product Rule
dx dx dx
= sin x(– sin x) + cos x (cos x)
= cos2 x – sin2 x = cos 2x

394,50 - Rajeev Gandhi Nagar Kota, Ph. No. : 93141-87482, 0744-2209671


IVRS No : 0744-2439051, 52, 53, www. motioniitjee.com , hr@motioniitjee.com

Downloaded from www.iitjeephysics4u.com


VECTOR & CALCULUS Page # 27

Rule No. 9 Derivatives Of Other Trigonometric Functions


Because sin x and cos x are differentiable functions of x, the related functions
sin x 1
tan x = ; sec x =
cos x cos x
cos x 1
cot x = ; cos ec x =
sin x sin x
are differentiable at every value of x at which they are defined. There derivatives, Calculated from the
Quotient Rule, are given by the following formulas.
d d
(tan x ) = sec 2 x ; (sec x ) = sec x tan x
dx dx

d d
(cot x ) = − cos ec 2 x ; (cos ec x ) = − cos ec x cot x
dx dx
Ex.42 Find dy / dx if y = tan x.
d d
cos x (sin x ) − sin x (cos x )
d d  sin x  dx dx
Sol. (tan x ) =  =
dx dx  cos x  cos 2 x
cos x cos x − sin x( − sin x ) cos 2 x + sin 2 x 1
= 2
= 2
= = sec 2 x
cos x cos x cos 2 x

d d
Ex.43 (a) (3x + cot x) = 3 + (cot x) = 3 – cosec2 x
dx dx

d  2  d d
(b) dx  sin x  = dx (2 cosec x ) = 2 dx (cosec x )
 
= 2(– cosec x cot x) = – 2 cosec x cot x

Rule No. 10 Derivative Of Logrithm And Exponential Functions


d 1 d x
(loge x ) = , (e ) = e x
dx x dx
Ex.44 y = ex . loge (x)

dy d x d dy ex
= (e ). log( x ) + [log e ( x )] e x ⇒ = e x . loge ( x ) +
dx dx dx dx x

Rule No. 11 Chain Rule Or ‘Outside Inside’ Rule


dy dy du
= .
dx du dx
It sometime helps to think about the Chain Rule the following way. If y = f (g(x)),
dy
= f’[g(x)] . g’(x)
dx
In words : To find dy/dx, differentiate the “outside” function f and leave the “inside” g(x) alone; then
multiply by the derivative of the inside.
We now know how to differntiate sin x and x2 – 4, but how do we differentiate a composite like
sin(x2 – 4)?
The answer is, with the Chain Rule, which says that the derivative of the composite of two differentiable
functions is the product of their derivatives evaluated at appropriate points. The Chain Rule is probably
the most widely used differentiation rule in mathematics. This section describes the rule and how to
use it. We begin with examples.

394,50 - Rajeev Gandhi Nagar Kota, Ph. No. : 93141-87482, 0744-2209671


IVRS No : 0744-2439051, 52, 53, www. motioniitjee.com , hr@motioniitjee.com

Downloaded from www.iitjeephysics4u.com


Page # 28 VECTOR & CALCULUS

Ex.45 The function y = 6x – 10 = 2(3x – 5) is the composite of the functions y = 2u and u = 3x – 5. How
are the derivatives of these three functions related ?

dy
Sol. We have = 6 , dy = 2 , du = 3
dx du dx
dy dy du
Since 6 = 2 × 3 = .
dx du dx
dy dy du
Is it an accident that = . ?
dx du dx
If we think of the derivative as a rate of change, our intution allows us to see that this relationship is
reasonable. For y = f(u) and u = g(x), if y changes twice as fast as u and u changes three times as fast
as x, then we expect y to change six times as fast as x.

Ex.46 Let us try this again on another function.


y = 9x4 + 6x2 + 1 = (3x2 + 1)2
is the composite y = u2 and u = 3x2 + 1. Calculating derivatives. We see that

dy du
. = 2u.6 x = 2 (3x2 + 1). 6x = 36x3 + 12 x
du dx

dy d
and = (9 x 4 + 6 x 2 + 1) = 36 x3 + 12 x
dx dx

dy du dy
Once again, . =
du dx dx
The derivative of the composite function f(g(x)) at x is the derivative of f at g(x) times the derivative of
g at x.

Ex.47 Find the derivation of y = x 2 + 1

2
Sol. Here y = f(g(x)), where f(u) = u and u = g(x) = x + 1. Since the derivatives of f and g are

1
f ′ (u) = and g′(x) = 2x,
2 u
the Chain Rule gives

dy d 1 1 x
= f (g( x )) = f′ (g(x)).g′(x) = .g′(x) = . (2x) =
dx dx 2 g( x ) 2 x2 + 1 2
x +1
derivative of
outside the outside

d
Ex.48
sin( x 2 + x ) = cos( x 2 + x ).(2x + 1)
dx
Inside Inside derivative
left along of the inside
d 1
Ex.49 (a) (1 – x 2 )1/ 4 = (1 – x 2 ) – 3 / 4 (–2x ) u = 1 – x2 and n = 1/4
dx 4
(Function defined) on [–1, 1]
–x
= (derivative defined only on (–1, 1))
2(1 – x 2 )3 / 4

394,50 - Rajeev Gandhi Nagar Kota, Ph. No. : 93141-87482, 0744-2209671


IVRS No : 0744-2439051, 52, 53, www. motioniitjee.com , hr@motioniitjee.com

Downloaded from www.iitjeephysics4u.com


VECTOR & CALCULUS Page # 29

d d
(b) sin 2x = cos 2x 2x = cos 2x .2 = 2 cos 2x
dx dx

d d
(c) ( A sin( ωt + φ)) = A cos (ω t + φ) (ω t + φ ) = A cos (ω t + φ). ω = A ω cos (ω t + φ)
dt dt

Rull No. 12 Power Chain Rule


d n du
* If u = nun –1
dx dx
d  1  d d
Ex.50   = (3 x – 2) –1 = – 1 (3x – 2)–2 (3 x – 2)
dx  3 x – 2  dx dx

3
= – 1 (3x – 2)–2 (3) = –
( 3 x – 2) 2
In part (d) we could also have found the derivation with the Quotient Rule.

d
Ex.51 (a) ( Ax + B)n
dx
du
Sol. Here u = Ax + B, =A
dx
d
∴ ( Ax + B )n = n( Ax + B)n –1.A
dx
d d 1
(b) sin( Ax + B) = cos( Ax + B).A (c) log(Ax + B) = .A
dx dx Ax + B
d d ( Ax +B )
(d) tan (Ax+B) = sec2 (Ax + B).A (e) e = e( Ax +B ) .A
dx dx
Note : These results are important

19. DOUBLE DIFFERENTIATION

If f is differentiable function, then its derivative f' is also a function, so f' may have a derivative of its
own, denoted by ( f ' )' = f ' ' . This new function f'' is called the second derivative of because it is the
derivative of the derivative of f. Using Leibniz notation, we write the second derivative of y = f(x) as

d  dy  d2 y
 =
dx  dx  dx 2

Another notation is f''(x) = D2 f (x).

Ex.52 If f(x) = x cos x, find f" (x)


d d
Sol. Using the Product Rule, we have f '(x) = x (cos x) + cos x ( x ) = – x sin x + cos x
dx dx
To find f" (x) we differentiate f'(x) :
d d d d
f"(x) = (– x sin x + cos x ) = – x (sin x ) + sin x (–x) + (cos x)
dx dx dx dx
= – x cos x – sinx – sinx = – x cos x – 2 sin x

394,50 - Rajeev Gandhi Nagar Kota, Ph. No. : 93141-87482, 0744-2209671


IVRS No : 0744-2439051, 52, 53, www. motioniitjee.com , hr@motioniitjee.com

Downloaded from www.iitjeephysics4u.com


Page # 30 VECTOR & CALCULUS

20. APPLICATION OF DERIVATIVE DIFFERENTIATION AS A RATE OF CHANGE


dy
is rate of change of 'y' with respect to 'x' :
dx
For examples :
dx
(i) v = this means velocity 'v' is rate of change of displacement 'x' with respect to time 't'
dt

dv
(ii) a = this means acceleration 'a' is rate of change of velocity 'v' with respect to time 't'.
dt

dp
(iii) F = this means force 'F' is rate of change of monentum 'p' with respect to time 't'.
dt

dL
(iv) τ = this means torque 'τ' is rate of change of angular momentum 'L' with respect to time 't'
dt

dW
(v) Power = this means power 'P' is rate of change of work 'W' with respect to time 't'
dt

π 2
Ex.53 The area A of a circle is related to its diameter by the equation A = D .
4
How fast is the area changing with respect to the diameter when the diameter is 10 m ?
Sol. The (instantaneous) rate of change of the area with respect to the diameter is
dA π πD
= 2D =
dD 4 2
When D =10m, the area is changing at rate (π/2) = 5π m2/m. This mean that a small change ∆D m in the
diameter would result in a changed of about 5p ∆D m2 in the area of the circle.

Physical Example :
Ex.54 Boyle's Law state that when a sample of gas is compressed at a constant temperature, the
product of the pressure and the volume remains constant : PV = C. Find the rate of change of
volume with respect to pressure.

dV C
Sol. =– 2
dP P

Ex.55 (a) Find the average rate of change of the area of a circle with respect to its radius r as r
changed from
(i) 2 to 3 (ii) 2 to 2.5 (iii) 2 to 2.1
(b) Find the instantaneous rate of change when r = 2.

(c) Show that thre rate of change of the area of a circle with respect to its radius (at any r) is
equal to the circumference of the circle. Try to explain geometrically when this is true by drawing
a circle whose radius is increased by an amount ∆r. How can you approximate the resulting
change in area ∆A if ∆r is small ?
Sol. (a) (i) 5π (ii) 4.5 π (iii) 4.1 π
(b) 4 π
(c) ∆A ≈ 2 π r ∆r

394,50 - Rajeev Gandhi Nagar Kota, Ph. No. : 93141-87482, 0744-2209671


IVRS No : 0744-2439051, 52, 53, www. motioniitjee.com , hr@motioniitjee.com

Downloaded from www.iitjeephysics4u.com


VECTOR & CALCULUS Page # 31

21. MAXIMA & MINIMA


y
Suppose a quantity y depends on another quantity x in a manner shown
in figure. It becomes maximum at x1 and minimum at x2. At these points
the tangent to the curve is parallel to the x-axis and hence its slope is
tan θ = 0. Thus, at a maxima or a minima slope
dy x
⇒ =0 x1 x2
dx

Maxima
Just before the maximum the slope is positive, at the maximum it
dy y
is zero and just after the maximum it is negative. Thus, decrease
dx
dy 3
at a maximum and hence the rate of change of is negative at θ2 2 4θ4
dx
d  dy  d  dy  θ11 5 θ5
a maximum i.e.,   < 0 at maximum. The quantity   is slope = m1 = tan θ1
dx  dx  dx  dx 
m1 > m 2 >(m3 = 0) > m4 > m5
the rate of change of the slope. It is written x
O For maxima, as x increases
d2 y dy d2 y
as =0 <0 the slope decreases
2 . Conditions for maxima are : (a) dx (b)
dx dx 2

Minima
Similarly, at a minimum the slope changes from negative to
positive, y slope = m1 = tan θ1
m1 < m2 <(m3 = 0) < m 4 < m5
Hence with the increases of x. The slope is increasing
θ1
that means the rate of change of slope with respect to x is θ2 5 θ5
positive. 1 θ4
2 4
d  dy  3 θ3 = 0
Hence  >0
dx  dx 

Conditions for minima are :


O x
For minima, as x increases
dy d2 y slope increases
(a) =0 (b) >0
dx dx 2
Quite often it is known from the physical situation whether the quantity is a maximum or a minimum.
d2 y
The test on may then be omitted.
dx 2
Ex.56 Find maximum or minimum values of the functions :
(A) y = 25x2 + 5 – 10x (B) y = 9 – (x – 3)2
dy
Sol. (A) For maximum and minimum value, we can put =0
dx
dy 1
or = 50 x – 10 = 0 ∴ x =
dx 5

d2 y
Further, = 50
dx 2

394,50 - Rajeev Gandhi Nagar Kota, Ph. No. : 93141-87482, 0744-2209671


IVRS No : 0744-2439051, 52, 53, www. motioniitjee.com , hr@motioniitjee.com

Downloaded from www.iitjeephysics4u.com


Page # 32 VECTOR & CALCULUS

d2 y 1 1
or 2 has positive value at x = . Therefore, y has minimum value at x = . Therefore, y has
dx 5 5

1 1
minimum value at x = . Substituting x = in given equation, we get
5 5
2
 1  1
ymin = 25  + 5 – 10  = 4
5 5

(B) y = 9 – (x – 3)2 = 9 – x2 – 9 + 6x
or y = 6x – x2
dy
∴ = 6 – 2x
dx
dy
For minimum or maximum value of y we will substitute =0
dx
or 6 – 2x = 0
x= 3
d2 y
To check whether value of y is maximum or minimum at x = 3 we will have to check whether is
dx 2
positive or negative.
d2 y
= –2
dx 2
d2 y
or is negative at x = 3. Hence, value of y is maximum. This maximum value of y is,
dx 2
ymax = 9 – (3 – 3)2 = 9

22. INTEGRATION
Definitions :
A function F(x) is a antiderivative of a function f(x) if
F'(x) = f(x)
for all x in the domain of f. The set of all antiderivatives of f is the indefinite integral of f with respect
to x, denoted by

∫ f ( x) dx
The symbol ∫ is an integral sign. The function f is the integrand of the integral and x is the variable of
integration.
For example f(x) = x3 then f'(x) = 3x2
So the integral of 3x2 is x3
Similarly if f(x) = x3 + 4
there for general integral of 3x2 is x3 + c where c is a constant
One antiderivative F of a function f, the other antiderivatives of f differ from F by a constant. We
indicate this in integral notation in the following way :

∫ f ( x) dx = F( x) + C .....(i)

The constant C is the constant of integration or arbitrary constant, Equation (1) is read, "The indefinite
integral of f with respect to x is F(x) + C." When we find F(x) + C, we say that we have integrated f
and evaluated the integral.

394,50 - Rajeev Gandhi Nagar Kota, Ph. No. : 93141-87482, 0744-2209671


IVRS No : 0744-2439051, 52, 53, www. motioniitjee.com , hr@motioniitjee.com

Downloaded from www.iitjeephysics4u.com


VECTOR & CALCULUS Page # 33

Ex.57 Evaluate ∫ 2x dx
an antiderivative of 2x

Sol.
∫ 2x dx = x
2
+C
the arbitrary constant
The formula x + C generatres all the antiderivatives of the function 2x. The function x2 + 1, x2 – π, and
2

x2 + 2 are all antiderivatives of the function 2x, as you can check by differentiation.
Many of the indefinite integrals needed in scientific work are found by reversing derivative formulas.

Integral Formulas
Indefinite Integral Reversed derivated formula

x n+1 d  x n+1 

n n
1. x dx = + C , n ≠ – 1, n rational dx  n + 1  = x
n +1

d
∫ dx = ∫ 1dx = x + C (special case)
dx
(x) = 1

cos kx d  cos kx 
2. ∫ sin kx dx = – k
+C –
dx  k 
 = sin kx

sin kx d  sin kx 
3. ∫ cos kxdx =
k
+C 
dx  k 
 = cos kx

d
∫ sec tan x = sec 2 x
2
4. xdx = tan x + C
dx
d
∫ cosec (– cot x ) = csc 2 x
2
5. xdx = – cot x + C
dx
d
6. ∫ sec x tan xdx = sec x + C dx
sec x = sec x tan x

d
7. ∫ cos ec x cot x dx = – cosec x + C dx
(– csc x ) = csc x cot x

Ex.58 Examples based on above formulas :

(a) ∫ dx = x + c
x6
(b) ∫ x 5 dx =
6
+C Formula 1 with n = 5

1
∫ ∫
dx = x –1/ 2 dx = 2x1/ 2 + C = 2 x + C 1
(c) Formula 1 with n = –
x 2

– cos 2x
(d) ∫ sin 2x dx = 2
+C Formula 2 with k = 2

x 1 sin(1/ 2)x x 1
(e) ∫ cos 2 dx = ∫ cos 2 xdx = 1/ 2
+C = ∫ 2 sin 2 + C Formula 3 with k =
2

394,50 - Rajeev Gandhi Nagar Kota, Ph. No. : 93141-87482, 0744-2209671


IVRS No : 0744-2439051, 52, 53, www. motioniitjee.com , hr@motioniitjee.com

Downloaded from www.iitjeephysics4u.com


Page # 34 VECTOR & CALCULUS

Ex.59 Right : ∫ x cos x dx = x sin x + cos x + C


Reason : The derivative of the right-hand side is the integrand :

d
Check : ( x sin x + cos x + C) = x cos x + sin x – sin x + 0 = x cos x.
dx

Wrong : ∫ x cos x dx = x sin x + C


Reason : The derivative of the right-hand side is not the integrand :

d
Check : ( x sin x + C) = x cos x + sin x + 0 ≠ x cos x
dx

Rule No. 1 Constant Multiple Rule


• A function is an antiderivative of a constant multiple k of a function f if and only if it is k times an
antiderivative of f.

∫ kf ( x)dx = k ∫ f ( x) dx
5( x )n +1
Ex.60 ∫ ∫
5 x n dx = 5 x n dx =
n+1
+c

Rule No.2 Sum And Difference Rule


A function is an antiderivative of a sum or difference f ± g if and only if it is the sum or difference of an
antiderivative of f an antiderivative of g.

∫ [f ( x) ± g( x)dx] = ∫ f ( x)dx ± ∫ g( x)dx


Ex.61 Term-by-term integration

∫ (x
2
Evaluate : − 2x + 5) dx

Sol. If we recognize that (x3/3) – x2 + 5x is an antiderivative of x2 – 2x + 5, we can evaluate the integral as

antideriva
 tive
 arbitrary constant
3
x
∫ ( x 2 − 2 x + 5 ) dx =
3
− x2 + 5x + C

If we do not recognize the antiderivative right away, we can generate it term by term with the sum
and difference Rule :

∫ (x ∫ ∫ ∫
2
− 2x + 5) dx = x 2 dx − 2xdx + 5dx

x3
= + C1 − x 2 + C 2 + 5 x + C 3
3
This formula is more complicated than it needs to be. If we combine C1, C2 and C3 into a single constant
C = C1 + C2 + C3, the formula simplifies to

x3
− x 2 + 5x + C
3

394,50 - Rajeev Gandhi Nagar Kota, Ph. No. : 93141-87482, 0744-2209671


IVRS No : 0744-2439051, 52, 53, www. motioniitjee.com , hr@motioniitjee.com

Downloaded from www.iitjeephysics4u.com


VECTOR & CALCULUS Page # 35

and still gives all the antiderivatives there are. For this reason we recommend that you go right to the
final form even if you elect to integrate term by term. Write

x3
∫ ∫
( x 2 − 2x + 5)dx = x 2 dx − 2 xdx + 5dx =∫ ∫ 3
− x 2 + 5x + C

Find the simplest antiderivative you can for each part add the constant at the end.

Ex.62 We can sometimes use trigonometric identities to transform integrals we do not know how to
evaluate into integrals. The inetgral formulas for sin2 x and cos2 x arise frequently in applications.

1 − cos 2x 1 − cos 2x
∫ sin ∫
2
(a) x dx = dx sin 2 x =
2 2

1 1 1
=
2 ∫
(1 − cos 2 x )dx =
2 ∫
dx −
2 ∫
cos 2x dx

x  1  sin 2x x sin 2x
+ −  +C = − +C
2  2 2 2 4

1 + cos 2x 1 + cos 2x
∫ cos ∫
2
(b) x dx = dx cos 2 x =
2 2

x sin 2x
= + + C As in part (a), but with a sign change
2 4

23. SOME INDEFINITE INTEGRALS (AN ARBITRARY CONSTANT SHOULD BE


ADDED TO EACH OF THESE INTEGRALS.

(ax + b )n +1 1
(a) ∫ (ax + b)n dx =
a(n + 1)
(provided n ≠ –1) + C (b) ∫ x dn = ln x + C
dx 1 1 ax
∫ a + bx = b ln(a + bx) + C ∫e
ax
(c) (d) dx = e +C
a

–1 1
(e) ∫ sin(ax + b) = a
cos(ax + b) + C ∫
(f) cos(ax + b ) =
a
sin(ax + b) + C

( 3 x + 2) 4 ( 3 x + 2) 4 2dx
Ex.63 (a) ∫ (3 x + 2)3 dx =
4×3
+C =
12
+C (b) ∫ x
= 2lnx + C

dx 1 dx 1
(c) ∫ 5 + 2x = 2 ln(5 + 2x) + C (d) ∫ 3 − 5x = − 5 ln(3 − 5x) + C
1
∫e ∫e
3x
(e) dx = e 3 x + C (f)
–x/2
dx = –2 e – x / 2 + C
3

1 1
(g) ∫ sin(3x + 5)dx = – 3 cos(3x + 5) + C (h) ∫ cos(2x − 5)dx = 2 sin(2x − 5) + C

394,50 - Rajeev Gandhi Nagar Kota, Ph. No. : 93141-87482, 0744-2209671


IVRS No : 0744-2439051, 52, 53, www. motioniitjee.com , hr@motioniitjee.com

Downloaded from www.iitjeephysics4u.com


Page # 36 VECTOR & CALCULUS

24. DEFINITE INTEGRATION OR INTEGRTION WITH LIMITS

Upper limit of integration The function is the integrand

x is the variable of integration


b
Integral sign

a 
f ( x ) dx

Lower limit of integration



Integral of f from a to b

∫ f ( x) dx = [g(x)]
b
a = g(b) − g(a)
a

4 4
Ex.64 ∫ –1 ∫
3dx = 3 dx = 3[ x ] 4–1 = 3 [4 – (–1)] = (3) (5) = 15
–1

π/2  π
∫ sin x dx = [– cos x] = – cos  + cos (0) = – 0 + 1 = 1
π/2
0
0 2

a a 5 5 b b
 x3  a3  x2  52 − 32  x5 / 2  2 5/2
∫ ∫ ∫x
2 3/2
Ex.65 (1) x dx =   = (2) xdx =   = =8 (3) dx =   = b
0  3 0 3 3  2 3 2 0  5 / 2 0 5

25. APPLICATION OF DEFINITE INTERGRAL


Calculation Of Area Of A Curve.

f(x)

dx

a x b x
From graph shown in figure if we divide whole area in infinitely small strips of dx width.

We take a strip at x position of dx width.

Small area of this strip dA = f(x) dx


b

So, the total area between the curve and x-axis = sum of area of all strips = ∫ f ( x)dx
a

Let f(x) ≥ 0 be continuous on [a,b]. The area of the region between the graph of f and the x-axis is


A = f ( x )dx
a

394,50 - Rajeev Gandhi Nagar Kota, Ph. No. : 93141-87482, 0744-2209671


IVRS No : 0744-2439051, 52, 53, www. motioniitjee.com , hr@motioniitjee.com

Downloaded from www.iitjeephysics4u.com


VECTOR & CALCULUS Page # 37

Ex.66 Using an area to evaluate a definite integral

Evaluate xdx ∫
a
0 < a < b. y

b
Sol. We sketch the region under the curve y = x, a ≤ x ≤ b (figure) and y=x
see that it is a trapezoid with height (b – a) and bases a and b.
The value
b
of the integral is the area of this trapezoid : a b
a+b b2 a2
Thus
a

xdx = (b – a ) •
2 =
2

2
a
0 a b x
5 2 2 a-b
( 5) (1)

1
xdx =
2

2
=2 The region in Example

and so on.

Notice that x2/2 is an antiderivative of x, further evidence of a connection between antiderivatives


and summation.

(i) To find impulse

dp
dF =
dt
so imples = ∫ F.dt
Ex.67 If F = kt then find impulse at t = 3 sec.
f
so impulse will be area under f - t curve

3 3
 t2 

I = kt dt = K  
0  2 0
t
9k
⇒ I=
2

2. To calculate work done by force :

 

w = f .dx

So area under f - x curve will give the value of work done.

394,50 - Rajeev Gandhi Nagar Kota, Ph. No. : 93141-87482, 0744-2209671


IVRS No : 0744-2439051, 52, 53, www. motioniitjee.com , hr@motioniitjee.com

Downloaded from www.iitjeephysics4u.com


Page # 38 VECTOR & CALCULUS

Exercise - I (only one option is correct)


SECTION - A : FUNCTION SECTION - B : DIFFERENTIATION OF ELEMENTRY
1. f(x) = cos x + sin x Find f(π/2) FUNCTIONS
Sol. F i nd the de ri vati ve of gi ve n func ti on w.r.t .
corresponding independent variable.
4. y = x 2 + x + 8
Sol.

5. s = 5t 3 − 3t 5
2. f(x) = 4x + 3 Find f(f(2))
Sol.
Sol.

6. y = 5 sin x
Sol.

3. f(x) = log x3 and g(x) = log x


Which of the following statement is / are true-
(a) f(x) = g(x) (b) 3f(x) = g(x)
(c) f(x) = 3g(x) (d) f(x) = (g(x))3
Sol.

7. y = x 2 + sin x
Sol.

394,50 - Rajeev Gandhi Nagar Kota, Ph. No. : 93141-87482, 0744-2209671


IVRS No : 0744-2439051, 52, 53, www. motioniitjee.com , hr@motioniitjee.com

Downloaded from www.iitjeephysics4u.com


VECTOR & CALCULUS Page # 39

8. y = tan x + cot x 12. y = sin x + cos x


Sol. Sol.

13. y = nx + e x
Find the first derivative & second derivative of given
functions w.r.t. corresponding independent variable. Sol.

9. y = 6 x 2 − 10 x − 5 x −2
Sol.

SECTION - C : DIFFERENTIATION BY PRODUCT RULE


Fi nd deri vat i ve of gi ven functi ons w.r.t. the
independent variable x.
12 4 1 14. x sin x
10. r = − +
θ θ3 θ 4 Sol.
Sol.

15. y = e x  nx
Sol.
11. ω = 3 z 7 − 7 z 3 + 21z 2
Sol.

394,50 - Rajeev Gandhi Nagar Kota, Ph. No. : 93141-87482, 0744-2209671


IVRS No : 0744-2439051, 52, 53, www. motioniitjee.com , hr@motioniitjee.com

Downloaded from www.iitjeephysics4u.com


Page # 40 VECTOR & CALCULUS

SECTION - D : DIFFERENTIATION BY QUOTIENT


16. y = ( x − 1) ( x 2 + x + 1)
RULE
Sol. Find derivative of given function w.r.t. the independent
variable.
sin x
19. y = cos x

Sol.

2  1
17. y = ( x + 1)  x + 5 + 
 x
Sol.
2x + 5
20. y =
3x − 2
Sol.

18. y = sin x cos x


nx
Sol. 21. y =
x
Sol.

394,50 - Rajeev Gandhi Nagar Kota, Ph. No. : 93141-87482, 0744-2209671


IVRS No : 0744-2439051, 52, 53, www. motioniitjee.com , hr@motioniitjee.com

Downloaded from www.iitjeephysics4u.com


VECTOR & CALCULUS Page # 41

SECTION - E : DIFFERENTIATION BY CHAIN RULE


t2 − 1
22. f ( t ) = , find f′(t) dy
t2 + t − 2 Find as a function of x
dx
Sol. 25. y = ( 2x + 1)5
Sol.

2x + 1
23. z =
x2 − 1 26. y = ( 4 − 3 x )9
Sol. Sol.

24. y = x 2 cot x  x
−7

27. y = 1 − 
Sol.  7 
Sol.

394,50 - Rajeev Gandhi Nagar Kota, Ph. No. : 93141-87482, 0744-2209671


IVRS No : 0744-2439051, 52, 53, www. motioniitjee.com , hr@motioniitjee.com

Downloaded from www.iitjeephysics4u.com


Page # 42 VECTOR & CALCULUS

−10
x  31. y = 2 sin(ωx + φ) where ω and φ constants
28. y =  − 1
2  Sol.
Sol.

SECTION - G : DIFFERENTIATION AS A RATE


29. y = sin 5 x MEASUREMENT
Sol. 32. Suppose that the radius r and area A = πr2 of a
circle are differentiable functions of t. Write an equation
that relates dA / dt to dr / dt.
Sol.

30. y = sin( x ) + ln( x 2 ) + e 2 x


Sol. 33. Suppose that the radius r and surface area S =
4πr2 of a sphere are differentiable functions of t. Write
ds dr
an equation that relates to .
dt dt
Sol.

394,50 - Rajeev Gandhi Nagar Kota, Ph. No. : 93141-87482, 0744-2209671


IVRS No : 0744-2439051, 52, 53, www. motioniitjee.com , hr@motioniitjee.com

Downloaded from www.iitjeephysics4u.com


VECTOR & CALCULUS Page # 43

SECTION - H : MAXIMA & MINIMA 37. y = sin u, u = 3 x + 1


34. Particle’s position as a function of time is given by
Sol.
x = − t 2 + 4t + 4 find the maximum value of position
coordinate of particle.
Sol.

38. y = 6u − 9, u = (1/ 2) x 4
35. Find the maximum and minimum values of function
Sol.
2x 3 − 15 x 2 + 36 x + 11
Sol.

SECTION - I x
39. y = cos u, u = −
dy 3
Given y = f(u) and u = g(x) Find
dx Sol.

36. y = 2u3 , u = 8 x − 1
Sol.

394,50 - Rajeev Gandhi Nagar Kota, Ph. No. : 93141-87482, 0744-2209671


IVRS No : 0744-2439051, 52, 53, www. motioniitjee.com , hr@motioniitjee.com

Downloaded from www.iitjeephysics4u.com


Page # 44 VECTOR & CALCULUS

(b) x–4
PART - II : INTEGRATION
Sol.
Find integrals of given functions
1. (a) 2x
Sol.

(c) x–4 + 2x + 3
Sol.

(b) x2
Sol.

1
3. (a)
x2
Sol.

(c) x2 – 2x + 1
Sol.

5
(b)
x2
Sol.

2. (a) –3x–4
Sol.

394,50 - Rajeev Gandhi Nagar Kota, Ph. No. : 93141-87482, 0744-2209671


IVRS No : 0744-2439051, 52, 53, www. motioniitjee.com , hr@motioniitjee.com

Downloaded from www.iitjeephysics4u.com


VECTOR & CALCULUS Page # 45

5 43
(c) 2 − 5. (a) x
x2 3
Sol. Sol.

1
(b) 3
3 x
3 Sol.
4. (a) x
2
Sol.

(c) 3 1
x+
3
x
3
(b) Sol.
2 x
Sol.

1 −1/ 2
6. (a) x
2
Sol.
1
(c) x+
x
Sol.

394,50 - Rajeev Gandhi Nagar Kota, Ph. No. : 93141-87482, 0744-2209671


IVRS No : 0744-2439051, 52, 53, www. motioniitjee.com , hr@motioniitjee.com

Downloaded from www.iitjeephysics4u.com


Page # 46 VECTOR & CALCULUS

1 −3 / 2 1
(b) − x 9.
2 3x
Sol. Sol.

Integrate by using the substitution suggested in


bracket.

(c) −
3 −5 / 2
x
10. ∫ sin 3x dx , (use, u = 3x)
2 Sol.
Sol.

11. ∫ sec 2t tan 2t dt , (use, u = 2t)


Sol.
7. (1 − x 2 − 3 x 5 )
Sol.

12. ∫ 5 dx
−2
Sol.
8. 3 sin x
Sol.

394,50 - Rajeev Gandhi Nagar Kota, Ph. No. : 93141-87482, 0744-2209671


IVRS No : 0744-2439051, 52, 53, www. motioniitjee.com , hr@motioniitjee.com

Downloaded from www.iitjeephysics4u.com


VECTOR & CALCULUS Page # 47

1
−1
∫e
π x
dx
13.

−4
2
dθ 17.
0
Sol.
Sol.

Use a definite integral to find the area of the region


4
between the given curve and the x-axis on the interval
x  [0, b]
14. ∫  2 + 3  dx
−2 18. y = 2x
Sol. Sol.

5 2 x
19. y = +1
15. ∫ r dr 2
2 Sol.
Sol.

Use a definite integral to find the area of the region


2π between the given curve and the x-axis on the interval
16. ∫ sin θ dθ
0
[0, π]
20. y = sin x
Sol. Sol.

394,50 - Rajeev Gandhi Nagar Kota, Ph. No. : 93141-87482, 0744-2209671


IVRS No : 0744-2439051, 52, 53, www. motioniitjee.com , hr@motioniitjee.com

Downloaded from www.iitjeephysics4u.com


Page # 48 VECTOR & CALCULUS

3. Rain is falling vertically down wards with a speed 5


PART - III VECTOR
SECTION - A : DEFINITION OF VECTOR & ANGLE m/s. If unit vector along upward is defined as ĵ ,
BETWEEN VECTORS represent velocity of rain in vector form.
   Sol.
1. Vectors A,B and C are shown in figure. Find angle
between
y

B 
45º A
30º
x
60º

C

     
(i) A and B (ii) A and C (iii) B and C .
Sol. 4. The vector joining the points A(1, 1, –1) and B(2, –
3, 4) & pointing from A to B is

(a) − î + 4 ĵ − 5k̂ (b) î + 4 ĵ + 5k̂

(c) î − 4 ĵ + 5k̂ (d) − î − 4 ĵ − 5k̂


Sol.

2. The forces, each numerically equal to 5 N, are acting


as shown in the Figure. Find the angle between forces? SECTION - B : ADDITION OF VECTORS
5. A man walks 40 m North, then 30 m East and then 40
m South. Find the displacement from the starting point?
5N Sol.

60º
5N

Sol.

394,50 - Rajeev Gandhi Nagar Kota, Ph. No. : 93141-87482, 0744-2209671


IVRS No : 0744-2439051, 52, 53, www. motioniitjee.com , hr@motioniitjee.com

Downloaded from www.iitjeephysics4u.com


VECTOR & CALCULUS Page # 49

   
6. Two forces F1 and F2 are acting at right angles to 9. Two vectors a and b inclined at an angle θ w.r.t.

each other, find their resultant ? each other have a resultant c which makes an angle
Sol.   
β with a . If the directions of a and b are interchanged,
then the resultant will have the same
(A) magnitude
(B) direction
(C) magnitude as well as direction
(D) neither magnitude nor direction.
Sol.

7. A vector of magnitude 30 and direction eastwards


is added with another vector of magnitude 40 and
direction Northwards. Find the magnitude and direction
of resultant with the east.
Sol.

 
10. Two vectors A and B lie in a plane. Another

vector C lies outside this plane. The resultant
  
A + B + C of these three vectors
(A) can be zero
  (B) cannot be zero
8. Two force of F1 = 500 N due east and F2 = 250 N  
  (C) lies in the plane of A + B
due north. Find F2 − F1 ?  
(D) lies in the plane of A − B
Sol.
Sol.

394,50 - Rajeev Gandhi Nagar Kota, Ph. No. : 93141-87482, 0744-2209671


IVRS No : 0744-2439051, 52, 53, www. motioniitjee.com , hr@motioniitjee.com

Downloaded from www.iitjeephysics4u.com


Page # 50 VECTOR & CALCULUS

11. The vector sum of the forces of 10 N and 6 N can  


14. The vector sum of two vectors A and B is
be
maximum, then the angle θ between two vectors is
(A) 2N (B) 8N (C) 18N (D) 20N
(A) 0º (B) 30º (C) 45º (D) 60º
Sol.
Sol.

    
15. Given : C = A + B . Also, the magnitude of A, B and
12. A set of vectors taken in a given order gives a 
closed polygon. Then the resultant of these vectors C are 12, 5 and 13 units respectively. The angle

is a between A and B is
(A) scalar quantity (B) pseudo vector π π
(A) 0º (B) (C) (D) π
(C) unit vector (D) null vector 4 2
Sol. Sol.

    
13. The vector sum of two force P and Q is minimum 16. If P + Q = P − Q and θ is the angle between P and
when the angle θ between their positive directions, is 
Q , then
π π π
(A) (B) (C) (D) π (A) θ = 0º (B) θ = 90º (C) P = 0 (D) Q = 0
4 3 2
Sol. Sol.

394,50 - Rajeev Gandhi Nagar Kota, Ph. No. : 93141-87482, 0744-2209671


IVRS No : 0744-2439051, 52, 53, www. motioniitjee.com , hr@motioniitjee.com

Downloaded from www.iitjeephysics4u.com


VECTOR & CALCULUS Page # 51

17. The sum and difference of two perpendicular 20. What are the x and the y components of a 25 m
vectors of equal lengths are displacement at an angle of 210º with the x-axis
(A) of equal lengths and have an acute angle between (clockwise) ?
them Sol.
(B) of equal lengths and have an obtuse angle between
them
(C) also perpendicular to each other and are of different
lengths
(D) also perpendicular to each other and are of equal
lengths
Sol.

21. One of the rectangular components of a velocity


of 60 km h–1 is 30 km h–1 . Find other rectangular
component ?
Sol.

SECTION - C : RESOLUTION OF VECTORS


18. Find the magnitude of 3 î + 2 ĵ + k̂ ?
Sol.

22. If 0.5 î + 0.8 ĵ + C k̂ is a unit vector. Find the value


of C
Sol.

19. If A = 3 î + 4 ĵ then find Â
Sol.

394,50 - Rajeev Gandhi Nagar Kota, Ph. No. : 93141-87482, 0744-2209671


IVRS No : 0744-2439051, 52, 53, www. motioniitjee.com , hr@motioniitjee.com

Downloaded from www.iitjeephysics4u.com


Page # 52 VECTOR & CALCULUS

23. The rectangular components of a vector are  


26. If | A | = 4 , | B | = 3 and θ = 60º in the figure. Find
(2, 2). The corresponding rectangular components of
   
( )
another vector are 1, 3 . Find the angle between the (a) A .B (b) | A × B |

two vectors. B
Sol.

θ 
A
Sol.

24. The x and y components of a force are 2N and –


3N. The force is

(A) 2 î − 3 ĵ (B) 2 î + 3 ĵ (C) − 2 î − 3 ĵ (D) 3 î + 2 ĵ


  
27. Three non-zero vectors A ,B & C satisfy the
Sol.
    
relation A .B = 0 & A . C = 0 . Then A can be parallel
to:
     
(A) B (B) C (C) B . C (D) B × C
Sol.

SECTION-D : PRODUCT OF VECTORS


 
25. If A = î + ĵ + k̂ and B = 2 î + ĵ find
   
(a) A .B (b) A × B
Sol.

28. The magnitude of scalar product of two vectors


is 8 and that of vector product is 8 3 . The angle
between them is
(A) 30º (B) 60º (C) 120º (D) 150º
Sol.

394,50 - Rajeev Gandhi Nagar Kota, Ph. No. : 93141-87482, 0744-2209671


IVRS No : 0744-2439051, 52, 53, www. motioniitjee.com , hr@motioniitjee.com

Downloaded from www.iitjeephysics4u.com


VECTOR & CALCULUS Page # 53

Exercise - II (One or more than one option correct)


SECTION - A : FUNCTION
4. y = 7 x + tan x
x −1 Sol.
1. If f ( x ) = then find f{f(x)}
x +1
Sol.

SECTION - C : DIFFERENTIATION BY PRODUCT RULE


Fi nd deri vat i ve of gi ven functi ons w.r.t. the
corresponding independent variable.

5. y = e x tan x
 x + 2, x < 2
2. If f ( x ) =  Evalute f(2), f(1), and f(3) Sol.
2x − 1, x ≥ 2

Sol.

6. y = x 2 sin 4 x + x cos −2 x

Sol.

SECTION - B : DIFFERENTIATION OF ELEMENTRY


FUNCTIONS
Find the first derivative and second derivative of given
functions w.r.t. the independent variable x.

3. y = nx 2 + sin x

Sol.  1  1 
7. y =  x +   x − + 1
 x  x 
Sol.

394,50 - Rajeev Gandhi Nagar Kota, Ph. No. : 93141-87482, 0744-2209671


IVRS No : 0744-2439051, 52, 53, www. motioniitjee.com , hr@motioniitjee.com

Downloaded from www.iitjeephysics4u.com


Page # 54 VECTOR & CALCULUS

SECTION - D : DIFFERENTIATION BY QUOTIENT


8. y = x 2 sin x + 2x cos x − 2 sin x
RULE
Sol. Find derivative of given functions w.r.t. the respective
independent variable.

sin x + cos x
11. y =
cos x
Sol.

9. y = x 2 cos x − 2x sin x − 2 cos x


Sol.

cot x
12. y =
1 + cot x
Sol.

10. r = (1 + sec θ) sin θ


Sol.

cos x x
13. y = x
+
cos x

Sol.

394,50 - Rajeev Gandhi Nagar Kota, Ph. No. : 93141-87482, 0744-2209671


IVRS No : 0744-2439051, 52, 53, www. motioniitjee.com , hr@motioniitjee.com

Downloaded from www.iitjeephysics4u.com


VECTOR & CALCULUS Page # 55

tan q 17. y = x( x 2 + 1) −1/ 2


14. p = 1 + tan q
Sol.
Sol.

dq
18. q = 2r − r 2 , find
SECTION - E : DIFFERENTIATION BY CHAIN RULE dr
Sol.
dy
Find as a function of x
dx

15. y = sin3 x + sin 3 x


Sol.

4
 x2 1
19. y =  +x− 
16. sin 2 ( x 2 + 1)  8 x 
Sol. Sol.

394,50 - Rajeev Gandhi Nagar Kota, Ph. No. : 93141-87482, 0744-2209671


IVRS No : 0744-2439051, 52, 53, www. motioniitjee.com , hr@motioniitjee.com

Downloaded from www.iitjeephysics4u.com


Page # 56 VECTOR & CALCULUS

SECTION - F : DIFFERENTIATION AS A RATE 22. A sheet of area 40 m2 in used to make an open


MEASUREMENT tank with a square base, then find the dimensions of
the base such that volume of this tank is maximum.
20. The radius r and height h of a circular cylinder are
related to the cylinder’s volume V by the formula Sol.
2
V = πr h.
(a) If height is increasing at a rate of 5 m/s while
radius is constant, Find rate of increase of volume of
cylinder.
(b) If radius is increasing at a rate of 5 m/s while
height is constant, Find rate of increase of volume of
cylinder.
(c) If height is increasing at a rate of 5 m/s and radius
is increasing at a rate of 5 m/s. Find rate of increase
of volume of cylinder.
Sol.
SECTION - H : MISCELLANEOUS
23. Find y′′ if
(a) y = cos x
Sol.

(b) y = sec x
Sol.

SECTION - G : MAXIMA & MINIMA


21. Find two positive numbers x & y such that x + y =
60 and xy is maximum.
Sol.

24. y = cos u, u = sin x


Sol.

394,50 - Rajeev Gandhi Nagar Kota, Ph. No. : 93141-87482, 0744-2209671


IVRS No : 0744-2439051, 52, 53, www. motioniitjee.com , hr@motioniitjee.com

Downloaded from www.iitjeephysics4u.com


VECTOR & CALCULUS Page # 57

∫x
25. y = sin u, u = x − cos x −3
4. ( x + 1) dx
Sol. Sol.

PART - II : INTEGRATION t t+ t
Find integrals of given functions
5. ∫ t2
dt

∫ (2x Sol.
3
1. − 5 x + 7) dx
Sol.

4+ t
6. ∫ t3
dt
1 2 
2. ∫  5 − x 3
+ 2x  dx

Sol.

Sol.

7. ∫ cos θ (tan θ + sec θ) dθ


3. ∫( x + 3 x dx ) Sol.
Sol.

394,50 - Rajeev Gandhi Nagar Kota, Ph. No. : 93141-87482, 0744-2209671


IVRS No : 0744-2439051, 52, 53, www. motioniitjee.com , hr@motioniitjee.com

Downloaded from www.iitjeephysics4u.com


Page # 58 VECTOR & CALCULUS

1
2π dx
8. ∫ θ dθ 11. ∫ 3x + 2
0
π

Sol. Sol.

3
7 Use a definite integral to find the area of the region
9. ∫
0
x 2 dx between the given curve and the x-axis on the interval
[0, b]
Sol. 12. y = 3x2
Sol.

10. ∫ cos x dx
0 13. y = b 2 − x 2
Sol. Sol.

394,50 - Rajeev Gandhi Nagar Kota, Ph. No. : 93141-87482, 0744-2209671


IVRS No : 0744-2439051, 52, 53, www. motioniitjee.com , hr@motioniitjee.com

Downloaded from www.iitjeephysics4u.com


VECTOR & CALCULUS Page # 59

PART - III : VECTOR 3. A vector is not changed if

SECTION - A : DEFINITION OF VECTOR & ANGLE (A) it is displaced parallel to itself


BETWEEN VECTORS (B) it is rotated through an arbitrary angle
 (C) it is cross-multiplied by a unit vector
1. Vector A points N–E and its magnitude is 3 kg ms–
1
it is multiplied by the scalar λ such that λ = –4 (D) it is multiplied by an arbitrary scalar
second. Find the direction and magnitude of the new Sol.
vector quantity. Does it represent the same physical
quantity or not ?
Sol.

4. Which of the arrangement of axes in fig. can be


labelled “right handed coordinate system” ? As usual,
each axis lable indicates the positive side of the axis.

2. A hall has the dimensions 10 m × 12 m × 14 m. A z


fly starting at one corner ends up at a diametrically
opposite corner. The magnitude of its displacement is
nearly (i) x (ii) x
(A) 16 m (B) 17 m
y z
(C) 18 m (D) 21 m y
Sol.
x
x

(iii) y (iv) z

z
y

x z

(v) y (vi) y

z x

(A) (i), (ii) (B) (iii) (iv)


(C) (vi) (D) (v)

394,50 - Rajeev Gandhi Nagar Kota, Ph. No. : 93141-87482, 0744-2209671


IVRS No : 0744-2439051, 52, 53, www. motioniitjee.com , hr@motioniitjee.com

Downloaded from www.iitjeephysics4u.com


Page # 60 VECTOR & CALCULUS

Sol. → → →
6. Find the resultant of three vectors OA, OB and OC
each of magnitude r as shown in figure ?
A

B
r
r
45º
45º
O C
r
Sol.

SECTION : B ADDITION OF VECTOR


 
5. The angle θ between directions of forces A and B
is 90º where A = 8 dyne and B = 6 dyne. If the
 
resultant R makes an angle α with A then find the
value of ‘ α ’ ?
Sol.

7. If the angle between two forces increases, the


magnitude of their resultant
(A) decreases
(B) increases
(C) remains unchanged
(D) first decreases and then increases
Sol.

394,50 - Rajeev Gandhi Nagar Kota, Ph. No. : 93141-87482, 0744-2209671


IVRS No : 0744-2439051, 52, 53, www. motioniitjee.com , hr@motioniitjee.com

Downloaded from www.iitjeephysics4u.com


VECTOR & CALCULUS Page # 61

8. A car is moving on a straight road due north with a  


10. When two vector a and b are added, the
uniform speed of 50 km h–1 when it turns left through
magnitude of the resultant vector is always
90°. If the speed reamins unchanged after turning,
(A) greater than (a + b)
the change in the velcoity of the car in the turning
(B) less than or equal to (a + b)
process is :
(C) less than (a + b)
(A) zero (D) equal to (a + b)
(B) 50 2kmh –1S − W direction Sol.
(C) 50 2kmh –1N − W direction

(D) 50 kmh –1 due West


Sol.

 
11. Given : A = 2 î + 3 ĵ and B = 5 î − 6 ĵ . The magnitude
9. Which of the following sets of displacements might  
of A + B is
be capable of bringing a car to its returning point ?
(A) 4 units (B) 10 units
(A) 5, 10, 30 and 50 km
(B) 5, 9, 9 and 16 km (C) 58 units (D) 61 units
(C) 40, 40, 90 and 200 km
Sol.
(D) 10, 20, 40 and 90 km
Sol.

394,50 - Rajeev Gandhi Nagar Kota, Ph. No. : 93141-87482, 0744-2209671


IVRS No : 0744-2439051, 52, 53, www. motioniitjee.com , hr@motioniitjee.com

Downloaded from www.iitjeephysics4u.com


Page # 62 VECTOR & CALCULUS

      
12. Given : A = 2 î − ĵ + 2k̂ and B = − î − ĵ + k̂ . The unit 14. Given : a + b + c = 0 . Cut of the three vectors a, b a
  
vector of A − B is nd c two are equal in magnitude. The magnitude of

3 î + k̂ 3 î k̂ − 3 î − k̂ the third vector is 2 times that of either of the two


(A) (B) (C) (D) having equal magnitude. The angles between the
10 10 10 10
vectors are
Sol. (A) 90º, 135º, 135º (B) 30º, 60º, 90º
(C) 45º, 45º, 90º (D) 45º, 60º, 90º
Sol.

    
13. If | A + B | = | A | = | B | , then the angle between A

and B is 15. Which of the following is a true statement ?
(A) 0 (B) 60º (C) 90º (D) 120º (A) A vector cannot be divided by another vector
Sol. (B) Angular displacement can either be a scalar or a
vector
(C) Since addition of vectors is commutative therefore
vector subtraction is also commutative
(D) The resultant of two equal forces of magnitude F
acting at a point is F if the angle between the two
forces is 120º.
Sol.

394,50 - Rajeev Gandhi Nagar Kota, Ph. No. : 93141-87482, 0744-2209671


IVRS No : 0744-2439051, 52, 53, www. motioniitjee.com , hr@motioniitjee.com

Downloaded from www.iitjeephysics4u.com


VECTOR & CALCULUS Page # 63

SECTION - C : RESOLUTION OF VECTORS 18. Six forces, 9.81 N each, acting at a point are
  coplanar. If the angles between neighbouring forces
16. If A = 3 î + 4 ĵ and B = î + ĵ + 2k̂ then find out unit
are equal, then the resultant is
 
vector along A + B . (A) 0 N (B) 9.81 N
(C) 2 × 9.81 N (D) 3 × 9.81 N
Sol.
Sol.


17. Vector A is of length 2 cm and is 60º above the SECTION : D PRODUCT OF VECTORS
  
x-axis in the first quadrant. Vector B is of length 2 19. If a = x1 î + y1 ĵ & b = x 2 î + y 2 ĵ . The condition that
cm and 60º below the x-axis in the fourth quadrant.  
  would make a & b parallel to each other is ______.
The sum A + B is a vector of magnitude.
(A) 2 along + y axis (B) 2 along + x-axis Sol.
(C) 1 along – x-axis (D) 2 along – x-axis
Sol.

 
20. A vector A points vertically downward & B points
 
towards east, then the vector product A × B is
(A) along west (B) along east
(C) zero (D) none of above
Sol.

394,50 - Rajeev Gandhi Nagar Kota, Ph. No. : 93141-87482, 0744-2209671


IVRS No : 0744-2439051, 52, 53, www. motioniitjee.com , hr@motioniitjee.com

Downloaded from www.iitjeephysics4u.com


Page # 64 VECTOR & CALCULUS

Exercise - III Subjective Level-I

1. Match the statements given in column-I with 2. Position of particle is given by S = t3 – 2t2 + 5t + 4
statements given in column-II
(a) Find the position of particle at t = 1 sec
Column - I Column - II
(b) Find the first derivative of S at t = 1 sec
    
(A) If | A | = | B | and | A + B | = | A | then (p) 90º (c) Find the second derivative of S t = 1 sec
  Sol.
angle between A and B is
(B) Magnitude of resultant of two (q) 120º
 
forces | F1 | = 8N and | F2 | = 4 N may be
 
(C) Angle between A = 2 î + 2 ĵ & B = 3 k̂ is (r) 12 N

(D) Magnitude of resultant of vectors (s) 14


 
A = 2 î + ĵ & B = 3 k̂ is
Sol.

 
3.Two forces F1 = 2 î + 2 ĵ N amd F2 = 3 î + 4 k̂ N are acting
on a particle
(a) Find the resultant force acting on particle
 
(b) Find the angle between F1 & F2
 
(c) Find the componant of force F1 along force F2

Sol.

394,50 - Rajeev Gandhi Nagar Kota, Ph. No. : 93141-87482, 0744-2209671


IVRS No : 0744-2439051, 52, 53, www. motioniitjee.com , hr@motioniitjee.com

Downloaded from www.iitjeephysics4u.com


VECTOR & CALCULUS Page # 65

4. Statement-1 : A vector is a quantity that has   


6. Statement-1 : If three vectors A,B and C satisfy
both magnitude and direction and obeys the triangle
    
law of addition. the relation A .B = 0 & A . C = 0 then the vector A is
Statement-2 : The magnitude of the resultant vector  
parallel to B × C .
of two given vectors can never be less than the
magnitude on any the given vector.    
State me nt-2 : A ⊥ B and A ⊥ C he nce A i s
(A) Statement-1 is True, Statement-2 is True;  
State me nt -2 i s a correc t ex pl anat i on for perpendicular to plane formed by B and C
Statement-1 (A) Statement-1 is True, Statement-2 is True;
(B) Statement-1 is True, Statement-2 is True; State me nt -2 i s a correc t ex pl anat i on for
Statement-2 is NOT a correct explanati on for Statement-1
Statement-1 (B) Statement-1 is True, Statement-2 is True;
Statement-2 is NOT a correct explanati on for
(C) Statement-1 is True, Statement-2 is False
Statement-1
(D) Statement-1 is False, Statement-2 is True (C) Statement-1 is True, Statement-2 is False
Sol. (D) Statement-1 is False, Statement-2 is True
Sol.

5. Statement-1 : If the rectangular components of a


force are 8 N and 6 N, then the magnitude of the force 7. Statement-1 : The minimum number of vectors of
is 10 N. unequal magnitude required to produce zero resultant
      is three.
Statement-2 : If | A | = | B | = 1 then | A × B |2 + | A .B |2 = 1 . Statement-2 : Three vectors of unequal magnitude
(A) Statement-1 is True, Statement-2 is True; which can be represeted by the three sides of a triangle
State me nt -2 i s a correc t ex pl anat i on for taken in order, produce zero resultant.
Statement-1 (A) Statement-1 is True, Statement-2 is True;
(B) Statement-1 is True, Statement-2 is True; State me nt -2 i s a correc t ex pl anat i on for
Stateent-2 i s N OT a correct expl anat i on for Statement-1
Statement-1 (B) Statement-1 is True, Statement-2 is True;
(C) Statement-1 is True, Statement-2 is False Statement-2 is NOT a correct explanati on for
Statement-1
(D) Statement-1 is False, Statement-2 is True
(C) Statement-1 is True, Statement-2 is False
Sol.
(D) Statement-1 is False, Statement-2 is True
Sol.

394,50 - Rajeev Gandhi Nagar Kota, Ph. No. : 93141-87482, 0744-2209671


IVRS No : 0744-2439051, 52, 53, www. motioniitjee.com , hr@motioniitjee.com

Downloaded from www.iitjeephysics4u.com


Page # 66 VECTOR & CALCULUS

8. Statement-1 : The angle between the two vectors 10. State true or false
     
π
(Î + Ĵ) and (k̂ ) is 2
radian. (i) If A & B are two force vectors A .B = B . A
Sol.
 
Statement-2 : Angle between two vectors A and B
 
 A .B 
is given by θ = cos −1 .
 AB 
 
(A) Statement-1 is True, Statement-2 is True;
State me nt -2 i s a correc t ex pl anat i on for
Statement-1
(B) Statement-1 is True, Statement-2 is True;
Statement-2 is NOT a correct explanation for
Statement-1      
(C) Statement-1 is True, Statement-2 is False (ii) If A & B are two force vectors then A × B = B × A
(D) Statement-1 is False, Statement-2 is True Sol.
Sol.

(iii) If the vector product of two non-zero vectors


vanishes, the vectors are collinear.
Sol.

9. Statement-1 : Distance is scalar quantity.


Statement-2 : Distance is the length of path
transversed.
(A) Statement-1 is True, Statement-2 is True;
State me nt -2 i s a correc t ex pl anat i on for
Statement-1
(B) Statement-1 is True, Statement-2 is True;
Statement-2 is NOT a correct explanation for (iv) If a function has maximum value at point P theh
Statement-1 slope of tangent drawn on function at point P is zero.
(C) Statement-1 is True, Statement-2 is False Sol.
(D) Statement-1 is False, Statement-2 is True
Sol.

394,50 - Rajeev Gandhi Nagar Kota, Ph. No. : 93141-87482, 0744-2209671


IVRS No : 0744-2439051, 52, 53, www. motioniitjee.com , hr@motioniitjee.com

Downloaded from www.iitjeephysics4u.com


VECTOR & CALCULUS Page # 67

11. Fill in the blanks (iv) The magnitude of area of the parallelogram formed
 
(i) The scalar product of vector A = 2 î + 5 k̂ and by the adjacent sides of vectors A = 3 î + 2 ĵ a nd
 
B = 3 ĵ + 5 k̂ is .......... B = 2 î − 2 k̂ is ......................
Sol. Sol.

 
(ii) If A = 3 î + 4 ĵ and B = 7 î + 24 ĵ , then the vector
 
having the same magnitude as B and parallel to A is
............ (v) A force i s represented by 2 î + 3 ĵ + 6 k̂ . The
Sol. magnitude of the force is .................
Sol.

    (v i) T he uni t ve ct or al ong v ec tor î + ĵ + k̂ is


(iii) If A || B then A × B = ..............
..................
Sol.
Sol.

394,50 - Rajeev Gandhi Nagar Kota, Ph. No. : 93141-87482, 0744-2209671


IVRS No : 0744-2439051, 52, 53, www. motioniitjee.com , hr@motioniitjee.com

Downloaded from www.iitjeephysics4u.com


Page # 68 VECTOR & CALCULUS

   
(vii) If A is ........................ to B , then A .B = 0
Sol.


(viii) The vector A = î + ĵ , where î and ĵ are unit
vectors along x-axis and y-axis respectively, makes
an angle of ..................... degree with x-axis.
Sol.

      
(ix) If A + B + C = 0 , then A .(B × C) = ....................
Sol.

394,50 - Rajeev Gandhi Nagar Kota, Ph. No. : 93141-87482, 0744-2209671


IVRS No : 0744-2439051, 52, 53, www. motioniitjee.com , hr@motioniitjee.com

Downloaded from www.iitjeephysics4u.com


VECTOR & CALCULUS Page # 69

Exercise - IV Subjective Level-II

1. If the resultant of two forces of magnitudes P and Q acting 3. A man moves towards 3m north then 4m towards east and
finally 5m towards 37º south of west. His displacement from
at a point at an angle of 60º is 7 Q , then P/Q is
origin is
(A) 1 (B) 3/2 (C) 2 (D) 4
(A) 5 2 m (B) 0 m (C) 1 m (D) 12 m
Sol.
Sol.

4. Three forces P, Q & R are acting at a point in the plane. The


2. The resultant of two forces F1 and F2 is P. If F2 is reversed,
angle between P & Q and Q & R are 150º & 120º respectively,
then resultant is Q. Then the value of (P2 + Q2) in terms of F1
then for equilibrium, forces P, Q & R are in the ratio
and F2 is
(A) 2(F12 + F22) (B) F12 + F22 (A) 1 : 2 : 3 (B) 1 : 2 : 3 (C) 3 : 2 : 1 (D) 3 :2:1
(C) (F1 + F2)2 (D) none of these Sol.
Sol.

394,50 - Rajeev Gandhi Nagar Kota, Ph. No. : 93141-87482, 0744-2209671


IVRS No : 0744-2439051, 52, 53, www. motioniitjee.com , hr@motioniitjee.com

Downloaded from www.iitjeephysics4u.com


Page # 70 VECTOR & CALCULUS

5. A man rows a boat with a speed of 18 km/hr in northwest 7. The resultant of two forces, one double the other in magni-
direction. The shoreline makes an angle of 15º south of west. tude is perpendicular to the smaller of the two forces. The angle
Obtain the component of the velocity of the boat along the between the two forces is
shoreline. (A) 150º (B) 90º (C) 60º (D) 120º
Sol.
3
(A) 9 km/hr (B) 18 km / hr
2
(C) 18 cos 15º km/hr (D) 18 cos 75º km/hr
Sol.

6. A brid moves from point (1, – 2, 3) to (4, 2, 3). If the speed of


the bird is 10m/sec, then the velocity vector of the bird is 8. If the angle between the unit vectors a and b is 60º, then

(A) 5( i − 2j + 3k ) (B) 5(4 i + 2j + 3k ) | a – b | is


(A) 0 (B) 1 (C) 2 (D) 4
(C) 0.6 i + 0.8 j (D) 6 i + 8 j Sol.
Sol.

394,50 - Rajeev Gandhi Nagar Kota, Ph. No. : 93141-87482, 0744-2209671


IVRS No : 0744-2439051, 52, 53, www. motioniitjee.com , hr@motioniitjee.com

Downloaded from www.iitjeephysics4u.com


VECTOR & CALCULUS Page # 71

9. For a particle moving in a straight line, the position of the      


11. Two vectors A and B are such that | A + B | = | A – B |. The
particle at time (t) is given by x = t3 – 6t2 + 3t + 7 what is the  
velocity of the particle when it’s acceleration is zero ? angle between the vectors A and B is -

(A) – 9ms–1 (B) –12ms–1 (C) 3ms–1 (D) 42ms–1 (A) 0 (B) π/3 (C) π/2 (D) π
Sol.
Sol.

10. Two forces each numerically equal to 10 dynes are acting 12. A particle moves through angular displacement θ on a cir-
as shown in the following figure, then their resultant is - cular path of radius r. The linear displacement will be -
(A) 2r sin (θ/2) (B) 2r cos (θ/2)
(C) 2r tan (θ/2) (D) 2r cot (θ/2)
10 dynes Sol.

60° 10 dynes

(A) 10 dynes (B) 20 dynes


(C) 10 3 dynes (D) 5 dynes
Sol.

394,50 - Rajeev Gandhi Nagar Kota, Ph. No. : 93141-87482, 0744-2209671


IVRS No : 0744-2439051, 52, 53, www. motioniitjee.com , hr@motioniitjee.com

Downloaded from www.iitjeephysics4u.com


Page # 72 VECTOR & CALCULUS

  
13. The vector P makes 120° with the x-axis and vector Q 15. The angle that the vector A = 2i + 3 j makes with y-axis is-
makes 30° with the y-axis. What is their resultant ? (A) tan–1(3/2) (B) tan–1(2/3)
(C) sin–1(2/3) (D) cos–1(3/2)
(A) P + Q (B) P – Q (C) P 2 + Q 2 (D) P 2 – Q 2
Sol.
Sol.

14. A man travels 1 mile due east, then 5 miles due south, then 16. A man moves towards 3m north then 4m towards east and
2 miles due east and finally 9 miles due north, how far is he finally 5 m towards 37° south of west. His
from the starting point - displacement from origin is -
(A) 3 miles (B) 5 miles (A) 5 2 m (B) 0 m (C) 12 m (D) 5 m
(C) 4 miles (D) between 5 and 9 miles
Sol.
Sol.

394,50 - Rajeev Gandhi Nagar Kota, Ph. No. : 93141-87482, 0744-2209671


IVRS No : 0744-2439051, 52, 53, www. motioniitjee.com , hr@motioniitjee.com

Downloaded from www.iitjeephysics4u.com


VECTOR & CALCULUS Page # 73


17. If 3 i + 2j + 8k and 2i + xj + k are at right angles that x= 19. If a is a vector and x is a non-zero scalar, then -
 
(A) 7 (B) –7 (C) 5 (D) –4 (A) x a is a vector in the direction of a
Sol.  
(B) x a is a vector collinear to a
 
(C) x a and a have independent directions
(D) none of these
Sol.

18. a1i + a2 j is a unit vector perpendicular to 4 i – 3 j if -


(A) a1 = .6, a2 = .8 (B) a1 = 3, a2 = 4
(C) a1 = .8, a2 = .6 (D) a1 = 4, a2 = 3
Sol.

394,50 - Rajeev Gandhi Nagar Kota, Ph. No. : 93141-87482, 0744-2209671


IVRS No : 0744-2439051, 52, 53, www. motioniitjee.com , hr@motioniitjee.com

Downloaded from www.iitjeephysics4u.com


Page # 74 VECTOR & CALCULUS

Exercise - V JEE-Problems
1.Two vectors have magnitudes 3 unit and 4 unit 2.When two forces of magnitude P and Q are perpen-
respectively. What should be the angle between them dicular to each other, their resultant is of magnitude
if the magnitude of the resultant is R. When they are at an angle of 180º to each other
(a) 1unit, R
Sol. their resultant is of magnitude . Find the ratio of P
2
and Q.
Sol.

(b) 5 unit and


Sol.

3.A body acted upon by 3 given forces is under equi-


(c) 7 unit. librium.
Sol. y

37°

F3
  
(a) If |F1| = 10 Nt.,|F2 | = 6 Nt. Find the values of |F3 | &
angle (θ).

394,50 - Rajeev Gandhi Nagar Kota, Ph. No. : 93141-87482, 0744-2209671


IVRS No : 0744-2439051, 52, 53, www. motioniitjee.com , hr@motioniitjee.com

Downloaded from www.iitjeephysics4u.com


VECTOR & CALCULUS Page # 75

Sol.

 5.A parti cl e i s act ed upon by t he force s


(b) Express F2 in unit vector form
  
Sol. F1 = 2i + aj − 3k , F2 = 5 i + cj − bk , F3 = bi + 5 j − 7k ,

F4 = c i + 6 j − ak . Find the values of the constants a,
b, c in order that the particle will be in equilibrium.
Sol.

4. If the four forces as shown are in equilibrium Ex-


 
press F1 & F2 in unit vector form.

15 N F2
°
30 10 N
37° 30°

F1

Sol.

394,50 - Rajeev Gandhi Nagar Kota, Ph. No. : 93141-87482, 0744-2209671


IVRS No : 0744-2439051, 52, 53, www. motioniitjee.com , hr@motioniitjee.com

Downloaded from www.iitjeephysics4u.com


Page # 76 VECTOR & CALCULUS

6.A plane body has perpendicular axes OX and OY  


8.(a) Calculate r = a − b + c where a = 5 i + 4 j − 6k ,
marked on it and is acted on by following forces
 
5P in the direction OY b = −2i + 2j + 3k and c = 4 i + 3 j + 2k .
4P in the direction OX Sol.
10P in the direction OA where A is the point (3a, 4a)
15P in the direction AB where B is the point (–a, a)
Express each force in the unit vector from & calculate
the magnitude & direction of sum of the vector of
these forces.
Sol.


(b) Calculate the angle between r and the z-axis.
Sol.

 
 (c) Find the angle between a and b
7. A vector A of length 10 units makes an angle of
 Sol.
60º with the vector B of length 6 units. Find the
 
magnitude of the vector difference A – B & the angle

it makes with vector A .
Sol.

394,50 - Rajeev Gandhi Nagar Kota, Ph. No. : 93141-87482, 0744-2209671


IVRS No : 0744-2439051, 52, 53, www. motioniitjee.com , hr@motioniitjee.com

Downloaded from www.iitjeephysics4u.com


VECTOR & CALCULUS Page # 77

ANSWER KEY Exercise-I


PART - I
SECTION - A
1. 1 2. 47 3. (c)
SECTION - B
dy ds dy dy
4. = 2x + 1 5. = 15 t 2 − 15 t 4 6. = 5 cos x 7. = 2x + cos x 8. sec 2 x − cos ec 2 x
dx dt dx dx

dy d2 y
9. = 12 x − 10 + 10 x −3 , = 12 − 30 x − 4
dx dx 2

dr d 2r
10. = −12 θ − 2 + 12 θ − 4 − 4θ −5 , = 24 θ −3 − 48 θ −5 + 20 θ − 6
dθ dθ 2

dω d2 ω
11. = 21z 6 − 21z 2 + 42 z , 2
= 126 z 5 − 42 z + 42
dz dz

dy d2 y dy 1 d2 y 1
12. = cos x − sin x, = − sin x − cos x 13. = + ex, = − 2 + ex
dx dx 2 dx x dx 2
x

SECTION - C
x dy 1
e
14. sin x + x cos x 15. e x nx + 16. = 3x 2 2
17. y′ = 3 x + 10 x + 2 −
x dx x2
18. cos 2 x − sin2 x
SECTION - D

−19 1 nx t 2 − 2t + 1
19. sec x 2
20. y′ = 21. − 22. f ′( t ) =
(3 x − 2) 2 x2 x2 ( t 2 + t − 2) 2

dz − 2x 2 − 2x − 2 dy
23. = 24. = − x 2 csc 2 x + 2x cot x
dx ( x 2 − 1)2 dx

SECTION - E

25. With u = (2x + 1)

dy dy
y = u5 : = = 5u 4 . 2 = 10( 2x + 1) 4
dx du

dy
26. = −27( 4 − 3 x )8
dx
−8
x dy dy du  1  x
27. With u = (1 −  ) y = u −7 : = = −7u −8 .  −  = 1 − 
7 dx du dx  7   7 
−11
dy x  2
28. = −5 − 1 29. 5 cos 5 x 30. cos( x ) + + 2e 2 x 31. 2ω cos(ωx + φ)
dx 2  x

394,50 - Rajeev Gandhi Nagar Kota, Ph. No. : 93141-87482, 0744-2209671


IVRS No : 0744-2439051, 52, 53, www. motioniitjee.com , hr@motioniitjee.com

Downloaded from www.iitjeephysics4u.com


Page # 78 VECTOR & CALCULUS

SECTION - G
dA dr ds dr
32. = 2πr 33. = 8πr
dt dt dt dt
SECTION - H
34. 8 35. ymax = 39, ymin = 38
SECTION - I
dy dy 1 x
36. = 48 (8 x − 1)2 37. 3 cos(3 x + 1) 38. 12x3 39. = − sin
dx dx 3 3

PART - II
x3 x3 −3 1 −3 1
1. (a) x 2 (b) (c ) − x 2 + x 2. (a)x (b) − x (c ) − x − 3 + x 2 + 3 x
3 3 3 3

1 5 5 2 x3
3. (a ) − (b) − (c ) 2x + 4. (a) x 3 (b) 3 x (c ) +2 x
x x x 3
2 4 2
x3 3x 3 3x 3 x3 x6
5. (a ) x 4/3
(b) (c ) + 6. (a) x1/ 2 (b) x −1/ 2 (c ) x −3 / 2 7. x − − +C
2 4 2 3 2

1 1
8. −3 cos x 9. nx 10. − cos 3 x + C
3 3

1 3π
11. sec 2t + C 12. 15 13. 14. Area = 21 15. 24 16. 0
2 2
b
b
∫ 2x dx = b
2
17. e – 1 18. Using n subintervals of length ∆x = and right-endpoint values : Area =
n 0

b2 b( 4 + b)
19. +b = 20. 2
4 4
PART - III
SECTION - A

1. (i) 105º, (ii) 150º, (iii) 105º 2. 120º 3. VR = −5 ĵ 4. (C)

SECTION - B

5. 30 m East 6. F12 + F22 7. 50, 53º with East 8. 250 5 N, tan −1 ( 2) W of N


9. (A) 10. (B) 11. (B) 12. (D) 13. (D) 14. (A)
15. (C) 16. (B) 17. (D)

SECTION - C
3 î + 4 ĵ
18. 14 19. 20. – 25 cos 30º and + 25 sin 30º 21. 30 3 km h −1
5
22. 0.11 23. 15º 24. (A)

SECTION - D
25. (a) 3 (b) − î + 2 ĵ − k̂ 26. (a) 6 (b) 6 3 27. (D) 28. (B)

394,50 - Rajeev Gandhi Nagar Kota, Ph. No. : 93141-87482, 0744-2209671


IVRS No : 0744-2439051, 52, 53, www. motioniitjee.com , hr@motioniitjee.com

Downloaded from www.iitjeephysics4u.com


VECTOR & CALCULUS Page # 79

ANSWER KEY Exercise-II


PART-I
SECTION-A
1
1. – 2. f(2) = 3, f(1) = 3, f(3) = 5
x
SECTION-B
6
– –13
dy 2 d2 y –2 7 d2 y –6
3. = + cos x , = – sin x 4. dy = x + sec 2 x ⇒ = × 7 +2 tan x sec 2 x
dx x dx 2 x2 dx 7 dx 2
49

SECTION-C
dy 2 1
5. ex(tanx + sec2x) 6. 2xsin4x + 4x2 sin3x cosx + cos–2x + 2xcos–3xsinx 7. = 1 + 2x + 3 – 2
dx x x
dy dr
8. x2 cosx 9. = – x 2 sin x 10. = cos θ + sec 2 θ
dx dθ

SECTION-D

dy – csc 2 x dy – x sin x – cos x x sin x + cos x sec 2 q


11. = sec 2 x 12. 13. = + 14.
dx (1 + cot x) 2 dx x2 cos 2 x (1 + tan q) 2
SECTION-E
1 1– r
15. 3sin2x cosx + 3cos3x 16. 4x sin (x2 + 1) cos (x2 + 1) 17. 2 3/2 18.
( x + 1) 2r – r 2
3
 x2   1 dy dy du x 1  x2 1  x 1
19. With u =  8  + x –   , y = u4 : = = 4u 3 .  + 1 + 2  = 4 , 4 + x –   + 1+ 2 
  x dx du dx  4   
x  8 x   4 x 

SECTION-F
dV dV dr dV dh dr
20. (a) = πr2 dh = 5 πr 2 (b) = 2πhr = 10 πrh (c) = πr 2 + 2πhr = 5πr2 + 10 πrh
dt dt dt dt dt dt dt
SECTION - G
40
21. x = 30 & y = 30 22. x= m
3
SECTION-H
23. (a) –cosx, (b) 2 sec3 x – sec x
dy
Given y = f(u) and u = g(x), find
dx
dy
24. – sin (sinx) cos x. 25. = cos (x – cosx) (1 + sinx)
dx
PART - II
x4 5 x2 x 1 2 3/2 3 4 /3 1 1 2
1. – + 7x + C 2. + + x2 + C 3. x + x +C 4. – – + C 5. 2 t – +C
2 2 5 x2 3 4 x 2x 2
t
2 –3/ 2 3π 2 7 1 5
6. –2t–2 – t +C 7. – cosθ + θ + C 8. 9. 10. 0 11. n
3 2 3 3 2
b
πb2
b
∫ 3x dx = b
2 3
12. Using n subintervals of length ∆x = and right-end point values : Area = 13.
n 0 4

394,50 - Rajeev Gandhi Nagar Kota, Ph. No. : 93141-87482, 0744-2209671


IVRS No : 0744-2439051, 52, 53, www. motioniitjee.com , hr@motioniitjee.com

Downloaded from www.iitjeephysics4u.com


Page # 80 VECTOR & CALCULUS

PART - III
SECTION - A
 
1. B = λA = –4 × 3 N − E = 12 S-W
No it does not represent the same physical quantity.
2. (D) 3. (A) 4. (A), (B), (C)
SECTION-B
5. 37° 6. r(1 + 2 ) 7. (A) 8. (B) 9. (B) 10. (B) 11. (C)
12. (A) 13. (D) 14. (A) 15. (A), (B), (D)
SECTION-C
4 i + 5 j + 2k
16. 17. B 18. (A)
45
SECTION- D
x1 y1
19. = 20. (D)
x2 y 2

ANSWER KEY Exercise-III

1. (A) → Q, (B) → R, (C) → P, (D) → S


   
F1.F2  3  F1.F2 6
2. (a) 8, (b) 4, (c) 2 (b) cos θ =   ⇒ θ = cos–1   (c) F1 cos θ =  =
|F1||F2 | 5 2 |F2 | 5
  
3. (a) FR = F1 + F2 = 2i + 5 j + 4k

4. (C) 5. (B) 6. (A) 7. (A) 8. (A) 9. (B)

10. (i) True (ii) False (iii) True (iv) True

1  1  1 
11. (i) 25 Units. (ii) 15i + 20 j (iii) Null vector (iv) 224 units (v) 7 units (vi) i+ j+ k
3 3 3
(vii) Perpendicular (viii) 45 (ix) zero.

ANSWER KEY Exercise-IV

1. C 2. A 3. B 4. D 5. A 6. D 7. D 8. B 9. A 10
A 11. C 12. A 13. A 14. B 15. B 16. B 17. B 18. A 19. B

ANSWER KEY Exercise-V


 
1. (a) 180°, (b) 90°, (c) 0 2. 2 ± 3 3. (a) |F3 | = 8 N, θ = 90° (b) F2 = –6 i
 
4. F1 = –(12 3 – 1) j & F2 = (12 – 5 3 ) i + (12 3 – 15) j 5. a = – 7, b = – 3, c = – 4

7
6. 5P ĵ,4P î,6Pî + 8P ĵ,–12P î – 9P ĵ , 20P, tan–1[–2] with the +ve x axis. 7. 2 19 ; cos–1
2 19

–1  –7  –1  –20 
8. (a) 11i + 5 j – 7k , (b) cos   , (c) cos  
 195   1309 

394,50 - Rajeev Gandhi Nagar Kota, Ph. No. : 93141-87482, 0744-2209671


IVRS No : 0744-2439051, 52, 53, www. motioniitjee.com , hr@motioniitjee.com

Downloaded from www.iitjeephysics4u.com

Você também pode gostar